#21 Rosh Review

Ace your homework & exams now with Quizwiz!

Question: What are the most common types of dementia?

Answer: Alzheimer's and vascular dementia. Rapid Review Dementia Gradual loss of mental capacity Preserved attention, motor function, speech Irreversible causes: Alzheimer's, vascular dementia, Creutzfeldt-Jakob disease Reversible causes: depression, B12 deficiency, syphilis, hypothyroidism, NPH, drug use, intracranial mass

Question: What post void residual capacity should initiate urology referral for incontinence?

Answer: Greater than 200 mL.

Question: What are some other, less common, diseases associated with optic neuritis?

Answer: Sarcoidosis and systemic lupus erythematous are associated optic neuritis and generally confer a poorer prognosis of visual recovery. Rapid Review Multiple Sclerosis Demyelinating disorder Caucasian females Optic neuritis (afferent pupillary defect, pain with eye movement, monocular vision loss, pale optic disc) Internuclear opthalmoplegia Lhermitte phenomenon: spinal electric shock sensation with neck flexion CSF: ↑ IgG protein, WBC pleocytosis Rx: methylprednisolone

Question: Which segment of the spine is most commonly involved in transverse myelitis?

Answer: Thoracic. The cervical spine is rarely affected. Rapid Review Transverse Myelitis Causes: viral, autoimmune, idiopathic Paraplegia, sensory impairment, sphincter disturbance Dx: MRI

Question: What is the treatment for herpes simplex?

Answer: Three medications can be used for treatment of herpes infections: acyclovir, valacyclovir, and famciclovir. Rapid Review Herpes Simplex Labialis Patient will be complaining of painful oral lesions PE will show painful vesicles and erosions on the tongue, buccal mucosa, and lips Labs will show multinucleated giant cells on Tzanck smear Diagnosis is made clinically. Gold standard is tissue culture with polymerase chain reaction (PCR) Most commonly caused by herpes simplex virus (HSV) type 1 Treatment is topical antiviral therapy or oral acyclovir

A 55-year old woman presents with feeling "swollen all over." You do not notice any signs of edema, despite her claim that her rings no longer fit and that she has gone from a shoe size of 6 to a 7 1⁄2 over the last 2 years. A review of systems reveals that she is sweating more than usual, feels fatigued, and often has a dull headache and diffuse arthralgias. On examination she has no joint swelling, erythema, tenderness and no skin abnormalities are noted. This presentation is most consistent with which one of the following? Acromegaly Cushing's disease Polymyalgia rheumatica Scleroderma

Correct Answer ( A ) Explanation: Acromegaly usually has an insidious onset, with a time to diagnosis on the order of 6-10 years. The classic facial findings, which include enlargement of the supraorbital ridges, mandible and a widened nose, are sometimes difficult to identify without a reference to the patient's appearance from several years earlier. Patients with this condition sometimes present with nonspecific symptoms such as arthralgias, fatigue, and headache. More specific complaints such as visual field defects or increasing ring and shoe sizes should prompt a physician to consider this diagnosis. The condition is caused by a growth hormone-secreting pituitary adenoma in 98% of cases. In younger patients this disease results in gigantism. Blood testing for growth hormone and IGF-1 is usually adequate for initial testing. Because of the relatively high incidence of nonfunctioning, incidentally discovered pituitary adenomas, imaging studies should be obtained only after a firm biochemical diagnosis of acromegaly has been made. The sella turcica should be imaged first, since GH-secreting pituitary adenoma is the most common cause of acromegaly. Magnetic resonance imaging is the imaging study of choice. For pituitary adenomas, transsphenoidal surgery is usually considered the first line of treatment, followed by medical therapy for residual disease. Somatostatin and dopamine analogues and growth hormone receptor antagonists are the mainstays of medical treatment for growth hormone excess after surgery. Clinical findings in Cushing's disease (B) include weight gain, truncal obesity, striae, hypertension, glucose intolerance and infections. The pituitary tumors in Cushing's disease are usually microadenomas, which, by definition, are 10 mm or less in diameter. These tumors are most often discovered when clinical manifestations from hypersecretion of ACTH prompt an appropriate diagnostic work-up. The classic clinical presentation of scleroderma (D) is a young or middle-age woman with Raynaud phenomenon and skin changes accompanied by musculoskeletal discomfort and gastrointestinal symptoms. Polymyalgia rheumatica (C), the most common chronic inflammatory condition in older adults, primarily affects proximal muscles and joints, causing disability. Proximal myalgias and arthralgias typically develop over weeks to months, worsening at night and with movement. True muscle weakness, as opposed to limited effort because of pain or atrophy from disuse, is not associated with pure polymyalgia rheumatica.

Question: What is the threshold for positivity of a PPD test placed on a 4-year-old girl?

Answer: 10 mm.

Question: At what age should breast cancer screening for women without risk factors stop?

Answer: 74 years.

Question: What is the recommended age for the administration of the Human Papillomavirus Virus (HPV) vaccine?

Answer: The Human Papillomavirus vaccine is recommended to all males and females aged 9-26 years.

Question: What is the diagnostic criteria used to diagnose osteoporosis?

Answer: A T-score less than -2.5 on DEXA.

Question: What is the membrane called that connects the middle ear to the cochlea?

Answer: The oval window.

Question: What is the role of corticosteroids in CNS abscess?

Answer: Corticosteroids should only be given to patients with cerebral edema related to the abscess. Rapid Review Intracranial Abscess Contiguous infection of middle ear, sinus, or teeth, surgery, trauma, hematogenous spread HA, fever, focal neurological deficit CT/MRI with contrast Ring enhancing lesions ABX (3rd gen cephalosporin + metronidazole if no source known), neurosurgery consultation

Question: What are the contraindications to using antenatal corticosteroid therapy?

Answer: Maternal systemic infections, namely chorioamnionitis. Rapid Review Infant (Neonatal) Respiratory Distress Syndrome ↓ Fetal surfactant Risk factors: prematurity (most common), maternal diabetes, c-section Respiratory difficulty within a few hours after birth CXR: ground glass appearance Rx: O2, intubation, CPAP, surfactant Concern for premature birth: prenatal glucocorticoids O2 complications: blindness, bronchopulmonary dysplasia

Question: What is a side effect of dapsone that can lead to hypoxia?

Answer: Methemoglobinemia. Rapid Review Pneumocystis Pneumonia (PCP) Patient with a history of HIV Complaining of gradual onset of non-productive cough Labs will show CD4 < 200, increased LDH CXR will show bilateral infiltrates (bat wing pattern) Most commonly caused by Pneumocystis jirovecii Treatment is TMP-SMX

Question: Which medications are the most powerful agents used in the treatment of gastroesophageal reflux disease?

Answer: Proton pump inhibitors. Rapid Review Gastroesophageal Reflux Disease (GERD) Patient with a history of nocturnal cough or asthma Complaining of retrosternal burning sensation radiating upward ("heartburn") usually after eating Diagnosis is made by empiric trial of PPIs Most commonly caused by LES dysfunction Treatment is weight loss, elevation head of bed during sleep, avoidance of certain foods (caffeine, alcohol, acidic foods)

Question: Which histopathologic term is commonly used to describe a gastric malignant cell?

Answer: Signet-ring cell, which may also be found in other GI, genital and breast cancers. Rapid Review Gastric Cancer Most common: adenocarcinoma Elderly males Most common risk factor: H. pylori Virchow nodes: metastasis to left supraclavicular node Sister Mary Joseph nodes: metastasis to umbilicus Krukenberg tumors: metastasis to ovaries Skin findings: acanthosis nigricans, Leser-Trélat sign

Question: What organism is classically associated with infectious endocarditis seen in patients with colon cancer?

Answer: Streptococcus bovis. Rapid Review Endocarditis Patient will be complaining of fever, rash, cough and myalgias PE will show Fever, Roth spots, Osler nodes, Murmur, Janeway lesions, Anemia, Nailbed hemorrhages, Emboli (FROM JANE) Diagnosis is made by echocardiography and Duke's criteria Most commonly caused by: IVDA: S. aureus, tricuspid Native valve: Streptococci, mitral Treatment is antibiotics Comments: GI malignancy: S. bovis

Question: If macrolide antibiotics are contraindicated in a patient with suspected pertussis, what is the second line alternative medication?

Answer: Trimethoprim/Sulfamethoxazole.

Which of the following HIV-positive patients suspected of having Pneumocystis pneumonia (PCP) should receive prednisone before treatment with trimethoprim/sulfamethoxazole? A 10-year-old with a normal chest x-ray and a PaO2 of 65 mm Hg A 15-year-old with diffuse interstitial infiltrates on chest x-ray, a pulse oximetry of 92% and PaO2 of 80 mm Hg A 20-year-old with diffuse interstitial infiltrates on chest x-ray and an A-a gradient of 25 mm Hg An 18-year-old with diffuse interstitial infiltrates on chest x-ray, a pulse oximetry of 94% on room air, PaO2 of 75 mm Hg, and an A-a gradient of 10 mm Hg

Correct Answer ( A ) Explanation: Corticosteroids are used as adjunct therapy in HIV-positive patients with severe PCP (now known as Pneumocystis jiroveci pneumonia), defined by a room air arterial oxygen partial pressure (PaO2) of less than 70 mm Hg or an alveolar-arterial oxygen gradient that exceeds 35 mm Hg. When administered, steroids should be given before trimethoprim/sulfamethoxazole or pentamidine because microbial degradation and clearance caused by antibiotics may trigger a severe inflammatory response. Corticosteroid therapy can blunt this inflammatory response, improve oxygenation, and reduce the incidence of respiratory failure. All of the other patients (B, C, & D) have either a room air arterial oxygen pressure (PaO2) greater than 70 mm Hg or an alveolar-arterial O2 gradient less than 35 mm Hg.

A 4-year-old girl presents to clinic for a well-child check and routine vaccination. History reveals that the patient and her mother have been residing in a homeless shelter for the past three months. Which of the following should be completed at this visit? Administration of all vaccines and placement of TST today Administration of all vaccines today, placement of the TST in 6 weeks Administration of inactivated vaccines and placement of the TST today Placement of the TST today, administration of all vaccines in 6 weeks

Correct Answer ( A ) Explanation: The above patient is due for her four-year immunizations, including MMR, Varicella, IPV, and DTaP. Both the MMR and Varicella vaccines contain live virus, while the IPV and DTaP vaccines do not. The patient also requires a tuberculin skin test (TST) due to her residence in a shelter, which is a risk factor for tuberculosis acquisition. Recent measles vaccination has been shown to decrease response to the TST. The effect of other live virus vaccines on the TST is less clear. Current recommendations state that the TST can be placed on the same day as administration of live virus vaccines. However, if not placed concurrently, the TST must be delayed 4-6 weeks from the time of live virus vaccination to ensure a reliable test. The above patient can receive all vaccinations and placement of the TST today. Administration of all vaccines today with placement of TST in 6 weeks (B) is incorrect because the TST can be administered on the same day as live virus vaccines so long as no other live virus vaccines have been administered in the past 4-6 weeks. Administration of inactivated vaccines and placement of the TST today (C) is incorrect because all vaccinations, live virus and inactivated, should be administered at this visit. Finally, placement of the TST today with administration of all vaccines in 6 weeks (D) is incorrect because placement of the TST should not prevent timely vaccine administration.

A 65-year-old Norwegian man presents with hematemesis. His past medical history includes alcohol abuse, tobacco use, Helicobacter pylori infection, heavy intake of smoked fish and diabetes mellitus. Endoscopic examination reveals a large pyloric mass. Which of the following is the most common cause of this type of gastric cancer? Diabetes mellitus Helicobacter pylori Smoked fish diet Tobacco use

Correct Answer ( B ) Explanation: Gastric cancer represents the fourth most common cancer worldwide. Adenocarcinoma is the main type, arising from the gastric glands, and accounting for near 95% of all gastric cancers. The remaining 5% is comprised of sarcomas and lymphomas. Risk factors include male gender, African-American or Asian race, age >70-years-old, family history, tobacco and alcohol use, salted, smoked, and nitrate-preserved foods, H.pylori infection (main cause, up to 65-80% of all gastric cancers), chronic gastritis, intestinal metaplasia and work exposure to coal and nickel dust, asbestos and rubber or timber processing. Symptoms include nausea, vomiting, heartburn, indigestion, loss of weight and appetite, bloating, central or upper abdominal pain, hematochezia (blood per rectum), hematemesis (vomiting of blood), melena (black tarry stool) and anemia associated fatigue. Metastatic spread may reveal palpable left supraclavicular or umbilical nodes (Virchow ad Sister Mary Joseph nodes, respectively). Diagnosis is confirmed with endoscopy with biopsies, but other modalities, like barium roentgenogram (barium swallow, or upper GI series) or computed tomography are also used. After the cancer has been staged, treatment options include surgery (most common), chemotherapy and radiotherapy. Surgical resection of the tumor and surrounding lymph nodes may be expanded to include a Billroth I or II removal of the pancreas and small intestine. Common in Japan and available in the US, endoscopic mucosal resection is used for early gastric cancer, and uses a wire loop to remove malignant tissue. Surgery is curative in 40% of cases, and if metastasis is present (80-90% of the time at diagnosis), may only serve as a palliative treatment. Prognosis is poor (5-year survival <5 - 15%) because most patients present with advanced disease. Diabetes mellitus (A) is not a direct cause of gastric cancer.Dietary factors are not proven causes of gastric cancer, but diets high in nitrate containing or smoked or pickled foods (C) are associated with a higher risk. Smoking (D) increases the risk of developing gastric cancer significantly, from 40-80 % over a non-tobacco user. However, gastric cancers due to tobacco use occur most commonly in the gastric cardia and gastroesophageal junction, not in the pylorus.

A 45-year-old woman presents to your office with complaints of worsening heartburn and food regurgitation that occur several times every week. Treatment with a proton pump inhibitor has not relieved her symptoms. Which of the following diagnostic studies is used to confirm the diagnosis? Computed tomography Endoscopy Magnetic resonance imaging Ultrasound

Correct Answer ( B ) Explanation: Gastroesophageal reflux is a normal physiologic occurrence that most people experience intermittently. The phenomenon becomes gastroesophageal reflux disease (GERD) when excessive amounts of gastric juice reflux into the esophagus causing uncomfortable symptoms or complications. A presumptive diagnosis of GERD can be made based on clinical symptoms of heartburn, food regurgitation and dysphagia. Additional diagnostic testing is used to confirm the diagnosis of GERD in patients not responding to therapy, to rule out other diagnoses or to detect complications of GERD. When diagnostic testing is indicated, the method of choice is upper endoscopy (EGD). This allows for the detection, management and stratification of the esophageal complications of GERD. Treatment strategies involve using either a step-up or step-down approach to therapy with lifestyle modifications, antacids, histamine 2 receptor antagonists, and proton pump inhibitors. There is no role for computed tomography (A), magnetic resonance imaging (C) or ultrasound (D) in the diagnostic workup of GERD.

A mother who is at 31 weeks gestation is at high risk of preterm delivery. Which of the following is given prenatally to prevent the risk of infant respiratory distress syndrome? Albumin Betamethasone Phosphatidylglycerol Sphingomyelin

Correct Answer ( B ) Explanation: Glucocorticoids, most commonly betamethasone, are administered to help prevent infant respiratory distress syndrome (hyaline membrane disease) in mothers with premature deliveries. The action of steroids increase the production of surfactant, the key substance which is lacking in newborns who suffer from this condition. Several organizations recommend treatment in women at risk of delivering before 34 weeks gestation. Lung maturity may be tested in women >30 weeks gestation. It is accomplished through the use of amniocentesis. Fluid samples are then sent for different laboratory studies, including the lecithin-to-sphingomyelin ratio, the surfactant-to-albumin ratio and phosphatidylglycerol levels. Albumin (A), phosphatidylglycerol (C) and sphingomyelin (D) are laboratory markers used in the detection of poor lung maturity, not in the prevention of infant respiratory distress syndrome.

A 46-year-old woman presents to the Emergency Department with fever, cough, and hemoptysis. She has a history of intravenous opioid use. Vital signs are BP 110/65 mm Hg, HR 120 beats per minute, RR 20 breaths per minute, and T 103.4°F. On auscultation of the chest, you hear a faint systolic ejection murmur. Which of the following is the most appropriate initial therapy? Ampicillin and gentamicin Cefepime and vancomycin Ceftriaxone and vancomycin Oxacillin and rifampin

Correct Answer ( B ) Explanation: This patient is presenting with signs and symptoms of infectious endocarditis. Risk factors for infectious endocarditis include rheumatic heart disease, congenital or acquired valvular disease, and intravenous drug use. Right-sided endocarditis involves either the pulmonic or tricuspid valve. It is classically seen in intravenous drug users. Organisms implicated in right-sided endocarditis include Staphylococcus aureus, Streptococcus pneumoniae, and gram negative bacteria. Presenting symptoms often include fever, cough, hemoptysis, chest pain, and dyspnea. Right-sided endocarditis is frequently misdiagnosed initially as pneumonia. Management includes antibiotics for the suspected organism based on the clinical situation. In an IV drug user, coverage should include methicillin-resistant Staphylococcus aureus and Pseudomonas aeruginosa in addition to the typically implicated organisms. The most appropriate antibiotics for this patient would include cefepime and vancomycin. Ampicillin and gentamicin (A) would be an appropriate antibiotic regimen for resistant Streptococcus viridans and non-resistant enterococci. It does not adequately cover MRSA, which should be covered in patients with IV drug use. Ceftriaxone and vancomycin (C) would not cover adequately for Pseudomonas aeruginosa, which should be covered in patients with IV drug use. Oxacillin and rifampin (D) would be partially appropriate for a patient with prosthetic valve endocarditis as it covers for Staphylococcus aureus and adds rifampin to penetrate the biofilm on prosthetic valves; however, gentamicin should be added in order to cover for Pseudomonas aeruginosa. This would also not be an adequate regimen for MRSA as oxacillin would need to be substituted with vancomycin.

A 14-year-old boy presents with headache, fever and altered mental status. He was recently treated for sinusitis 1 week ago. A noncontrast CT scan of the head is performed. What management is indicated? Ceftriaxone, metronidazole and admit Ceftriaxone, metronidazole and neurosurgery consultation Ceftriaxone, vancomycin and neurosurgery consultation Pyramethamine, sulfadiazine and admit

Correct Answer ( B ) Explanation: This patient presents with signs, symptoms and imaging consistent with an intracranial abscess. Immediate management involves administration of antibiotics covering the most likely pathogens and neurosurgical consultation. CNS abscess is unusual in immunocompetent hosts but can result from direct spread from other infections. The most common causes from direct spread are dental infections, otitis media and sinus infections. Patients with multiple abscesses should raise suspicion for endocarditis. Streptococcus species and anaerobic bacteria are the predominant causative agents. Patients with a history of trauma or intracranial surgery are also at risk for MRSA. CT scan is the diagnostic modality of choice and typically is followed by a lumbar puncture (unless there are signs of increased intracranial pressure), which aids in determining the etiologic agent. Location, size and number of abscesses dictate management. Patients with a single abscess are more readily treated with neurosurgical intervention while those with multiple abscesses are usually treated with antibiotics alone. Initial antibiotics in patients without a history of neurosurgery should be with a third generation cephalosporin and an agent covering anaerobic bacteria like metronidazole. Patients with CNS abscesses should not be admitted (A) without neurosurgical consultation as many will require emergent surgery. Ceftriaxone and vancomycin (C) is inadequate coverage of the typical bacterial agents. Pyramethamine and sulfadiazine (D) are typically used in the treatment of CNS toxoplasmosis, which appears as ring enhancing lesions on CT scan.

Screening for osteoporosis should be performed in which one of the following groups? 55-year old man with type 2 diabetes mellitus 55-year old woman with a BMI 30 kg/m2 65-year old previously healthy woman 75-year old previously healthy man

Correct Answer ( C ) Explanation: According to the United States Preventative Task Force (USPTF), all women 65 years or older and at-risk women aged 50-64 should undergo universal screening for osteoporosis. There is no evidence for screening men. For women aged 50-64, the presence of factors associated with low bone density would merit screening. Risk factors include low body weight, previous fracture, a family history of osteoporosis with fracture, a history of falls, physical inactivity, low vitamin D or calcium intake, and the use of certain medications or the presence of certain medical conditions. Screening allows asymptomatic patients with osteoporosis to be identified and treated. The bone measurement tests most commonly used to screen for osteoporosis are DEXA of the hip and lumbar spine, and quantitative ultrasonography of the calcaneus. Quantitative ultrasonography is less expensive and more portable than DEXA, does not expose patients to ionizing radiation, and can feasibly be implemented in primary care settings. Quantitative ultrasonography of the calcaneus predicts fractures of the femoral neck, hip, and spine as effectively as DEXA. However, current diagnostic and treatment criteria for osteoporosis rely on DEXA measurements only, and quantitative ultrasonography measurements are not interchangeable with DEXA measurements. The USPSTF recommends against daily supplementation with 400 IU or less of vitamin D3 and 1,000 mg or less of calcium for the primary prevention of fractures in noninstitutionalized, postmenopausal women. The USPSTF concluded that the current evidence is insufficient to assess the balance of benefits and harms of screening for osteoporosis in men, regardless of age. Therefore, a 55-year old man with type 2 diabetes mellitus (A) and a 75-year old previously healthy man (D) should not undergo screening for osteoporosis. Women whom are younger than 65 years of age with a BMI of 30kg/m2 (B) should not undergo screening for osteoporosis because they are not at increased risk. In fact, obesity is a protective factor of osteoporosis due to the increase in peripheral estrogen. These women are however at increased risk for osteoarthritis.

A two-month-old boy is sent to the emergency department by his pediatrician for cough and an abnormal CBC with lymphocytosis. He is up-to-date with immunizations. His older brother, who is four years old, is not up-to-date since the pediatrician suspended his immunizations due to a developing neurologic condition. The brother has also had a febrile illness and has been coughing for more than three weeks. You observe the two-month-old coughing and see a period of peri-oral cyanosis. What is the most sensitive and specific method of confirming your suspected diagnosis? Blood cultures Direct fluorescent antibody on nasal swabs Polymerase chain reaction of nasopharyngeal secretions Sputum cultures

Correct Answer ( C ) Explanation: The patient has pertussis. Pertussis is a highly contagious respiratory infection caused by Bordetella pertussis. Pertussis is commonly called whooping cough, secondary to the sound that is made on inspiration immediately following a paroxysm of coughing. There has been a recent resurgence of pertussis in the United States. This resulted in a revision of the booster schedule to require patients older than 10 years to obtain Tdap rather than tetanus (Td) alone. Transmission is via respiratory droplets and has an incubation of seven to 10 days before symptoms begin. Pertussis is a three-phase illness with each phase lasting approximately one to two weeks. First, there is the catarrhal phase with symptoms similar to an upper respiratory infection. Second, is the paroxysmal phase. This includes harsh coughing paroxysms lasting minutes. Finally, the third phase is the convalescent phase. This last phase demonstrates clinical recovery but, unfortunately, the patient remains infectious. Symptoms include persistent cough, posttussive emesis, apnea, cyanosis, fever, and inspiratory whoop. Lab work may reveal an absolute lymphocytosis. The most sensitive and specific testing is via polymerase chain reaction (PCR) of nasopharyngeal secretions. Blood cultures (A) often identify a source in pneumonia but not pertussis. Nasal swabbing with antibody testing (B) is highly specific but less sensitive than PCR. Sputum cultures (D) are also highly specific but not very sensitive as pertussis is highly fastidious and difficult to isolate and grow via culture.

A 23-year-old woman presents to her primary care physician for a routine annual visit. Her last Papanicolaou test was four years ago. She is a current smoker and is sexually active. Her mother was diagnosed with ovarian cancer at age 50. Which of the following should be offered to the patient? Mammography Meningococcal vaccine Ovarian ultrasound Papanicolaou test

Correct Answer ( D ) Explanation: Routine screening for cervical cancer with the Papanicolaou test is recommended for all women aged 21 years or older, at least every three years. For women aged 30 to 65 years who want to lengthen their screening interval, screening with a combination of cytology and human papillomavirus (HPV) testing every 5 years is recommended. These routine screening recommendations do not apply to women who have had cervical cancer or a high-grade precancerous lesion. This patient should also be counseled on smoking cessation and high-risk sexual behavior. The United States Preventative Task Force recommends biennial screening mammography (A) for women aged 50-74 years. Increasing age is the most important risk factor for breast cancer in women. The meningococcal vaccine (B) is a routine vaccination series given throughout childhood to prevent meningitis. It is also recommended for college students up to 21 years of age who are living in residence halls if they did not receive the vaccine at 16 year of age or older. There is no screening test for ovarian cancer (C). An ultrasound of the ovaries could be beneficial for an infertility workup, an ovarian torsion or an ectopic pregnancy.

A 15-year-old girl presents for increased urinary frequency and vaginal discharge. On pelvic exam you note frothy yellow discharge and a "strawberry cervix." Which of the following is the most likely diagnosis? Bacterial vaginosis Candidiasis Chlamydia Trichomoniasis

Correct Answer ( D ) Explanation: Trichomoniasis is one of the most common sexually transmitted infections. The flagellated organism, Trichomonas vaginalis, causes it. Symptoms in women include vaginal burning and itching, abnormal vaginal odor, and dyspareunia. Although men can contract the disease, they are often asymptomatic. On exam the vulva may be edematous, excoriated, and erythematous while the vagina and cervix may be red and inflamed. The characteristic finding on exam is a "strawberry cervix" which is caused by friable mucosa with punctate hemorrhagic ulcerations. Microscopy of vaginal secretions reveals flagellated organisms. Treatment is with metronidazole. Partners should also be treated. Bacterial vaginosis (A) is a polymicrobial infection (Gardnerella vaginalis is thought to be most common bacteria) that is associated with the use of anything that disrupts the normal balance of vaginal flora, such as antibiotics or IUDs. It is associated with a fishy, amine odor and has characteristic clue cells on wet mount. Treatment is with metronidazole. Candidasis (B) presents with pruritus and cottage cheese-like discharge. The diagnosis is made by microscopic examination showing germinated yeast (pseudohyphage). Chlamydia (C) is associated with mucopurulent discharge associated with cervicitis and urethritis. It commonly occurs with gonorrhea so treatment should include antibiotics for both infections (ceftriaxone for gonorrhea and azithromycin for Chlamydia).

Question: What is the treatment of a pregnant patient with syphilis and a penicillin allergy?

Answer: Admission for penicillin desensitization as only penicillin is effective in these patients. Rapid Review Syphilis Primary: painless chancre Secondary: lymphadenopathy, comdyloma lata, rash on palms/soles Tertiary: gummas VDRL and RPR positive 4-6 weeks after infection Primary/secondary: IM benzathine penicillin G x 1 dose Tertiary: IM benzathine penicillin G x 3 weeks

Question: Which serum component, if low, may cause a spuriously low total serum calcium reading?

Answer: Albumin concentrations lower than 4 g/dL will reduce the total serum calcium level. Rapid Review Hypercalcemia Malignancy (most common inpatient cause) Primary hyperparathyroidism (most common outpatient cause) Bones, stones, groans, psychiatric overtones ↓ QT interval Initial rx: IVF

Question: What are the clinical manifestations of hyperprolactinemia?

Answer: Although hyperprolactinemia in females occurs as the common triad of amenorrhea, galactorrhea and infertility, it can occur in males too, but presents with infertility, decreased libido and rarely, galactorrhea. Rapid Review Pituitary Tumors Visual sx (bitemporal hemianopia) Prolactinoma: MC, amenorrhea, impotence Growth hormone tumor: gigantism (children), acromegaly (adults) ACTH secreting TSH secreting Nonfunctioning

Question: What is the normal width of the retropharyngeal space on lateral neck X-ray?

Answer: 7 mm (at C2) in both children and adults. Retropharyngeal Abscess 3-5 years of age S. aureus, GAS, anaerobes, foreign body Sore throat, dysphagia Toxic appearing, drooling, ↓ neck extension, fever Cri du canard (duck "quack") Neck x-ray: widened retropharyngeal space twice the size of the vertebral body CT Rx: IV ABX, ENT consultation

Question: What is the most common cause of an acute hemarthrosis after a sports-related knee injury?

Answer: ACL tear. Rapid Review Patellofemoral Pain Syndrome Patient will be a women Complaining of aching anterior knee pain, that is worse with loaded flexion (stair climbing, jumping, prolonged sitting) PE will show patellar crepitation Treatment is to strengthen medial quadriceps, NSAIDs

Question: In type 1 diabetes the C peptide level should be absent, normal or elevated?

Answer: Absent. Rapid Review Diabetes Mellitus Type 2 ↑ Insulin resistance Obesity, family hx Candidiasis Blurry vision Hyperosmolar hyperglycemic state Lifestyle changes First line pharmaceutical rx: metformin

Question: What is the difference between acromegaly and gigantism?

Answer: Acromegaly occurs after the growth plate cartilage fuses in adulthood, whereas gigantism occurs before the growth plate cartilage fuses. Rapid Review Acromegaly Patient will be complaining of increased head, glove, or shoe size PE will show coarse facial features, oily skin, visual field deficits, diabetes Labs will show increased IGF-1 Most commonly caused by pituitary adenoma Treatment is transphenoid resection

Question: Approximately what percent of patients with an acute MI who present to the ED are discharged from the ED without a diagnosis?

Answer: Approximately 2%-4%. Rapid Review Ischemic Heart Disease #1 cause of death in USA RFs: family hx, smoking, HTN, DM, cholesterol, male, age >55 Stable angina: activity → chest pain (CP), relieved by rest, NTG Unstable angina: CP at rest CP, dyspnea, diaphoresis, nausea, hiccups, radiation to shoulder/jaw/back Elderly, diabetics, females, hx of stroke or HF: ↑ risk for atypical presentation Earliest ECG sign of MI: hyperacute T waves Up to 50% of ECGs are negative or nonspecific Highest S/S: troponin I

Question: What laboratory measures of coagulation are abnormal in vWD?

Answer: Bleeding time is increased, vWF activity and Factor VIII activity are decreased. Rapid Review von Willebrand Disease Patient with a history of a parent with similar symptoms Complaining of mucosal hemorrhage or bleeding that is difficult to control Labs will show decreased factor VIII, prolonged bleeding time Treatment is desmopressin (DDAVP) Comments: Most common inherited bleeding disorder, autosomal dominant

Question: If MRI is not available, what other radiographic modality may identify cord compression?

Answer: CT Myelography. Rapid Review Prostate Cancer RFs: age (most important), African-Americans, family hx Asymptomatic until advanced Obstructive uropathy Back pain: metasteses to lumbar spine (↑ ALP) PSA > 10 ng/mL Dx: needle core biopsy

Question: Which cardiac biomarker has the highest sensitivity and specificity?

Answer: Cardiac troponin. Rapid Review Ischemic Heart Disease #1 cause of death in USA RFs: family hx, smoking, HTN, DM, cholesterol, male, age >55 Stable angina: activity → chest pain (CP), relieved by rest, NTG Unstable angina: CP at rest CP, dyspnea, diaphoresis, nausea, hiccups, radiation to shoulder/jaw/back Elderly, diabetics, females, hx of stroke or HF: ↑ risk for atypical presentation Earliest ECG sign of MI: hyperacute T waves Up to 50% of ECGs are negative or nonspecific Highest S/S: troponin I

Question: What is the most common cause for secondary hyperparathyroidism?

Answer: Chronic Kidney disease causing decreased levels of 1,25-dihydroxyvitamin D.

Question: What laboratory abnormalities are seen in hemolysis?

Answer: Decreased hemoglobin, decreased haptoglobin, elevated LDH, elevated unconjugated bilirubin, schistocytes on peripheral blood smear.

Question: What are some transient causes of urinary incontinence commonly seen in the geriatric population?

Answer: Delirium, infection, atrophic vaginitis, stool impaction, medications, restricted mobility

Question: Name some organic causes of altered intestinal motility?

Answer: Diabetic autonomic neuropathy, gastroparesis, hyperthyroidism, scleroderma, amyloidosis. Rapid Review Irritable Bowel Syndrome Patient will be a woman With a history of constipation alternating with diarrhea Complaining of abdominal discomfort which is relieved with bowel movements Diagnosis is made by Rome criteria Treatment is symptomatic - dietary management and drugs

Question: Mixing alcohol and metronidazole leads to what reaction?

Answer: Disulfram-like reaction: flushing, tachycardia, hypotension. Rapid Review Bacterial Vaginosis Patient will be complaining of malodorous vaginal discharge PE will show thin, gray/white discharge Labs will show pH > 4.5, clue cells Diagnosis is made by KOH to smear → fishy odor, "whiff test", Amsel Criteria Most commonly caused by Gardnerella vaginalis Treatment is metronidazole

Question: What is the name of the rash associated with Lyme disease?

Answer: Erythema migrans—described as a circular lesion with central clearing or erythema at the site of a tick bite. Rapid Review Lyme Disease NE USA Campers/hikers Ixodes tick harbors Borrelia burgdorferi Stage I: erythema migrans (pathognomonic), viral-like syndrome Stage II: arthritis, myocarditis, bilateral Bell's palsy Stage III: chronic arthritis, chronic encephalopathy Rx: doxycycline CNS/cardiac involvement: IV ceftriaxone Pregnant/children rx: amoxicillin

Question: What is Conn's syndrome?

Answer: Excess aldosterone produced by an adrenal adenoma. Rapid Review Primary Adrenal Insufficiency (Addison's Disease) Patient will be complaining of abdominal pain, nausea, vomiting, diarrhea, fever, and confusion PE will show hyperpigmentation of skin and mucus membranes and hypotension Labs will show hyponatremia and hyperkalemia Most commonly caused by autoimmune Treatment is hydrocortisone

Question: What confirmatory test should be done next if a patient has a positive RPR?

Answer: FTA-ABS test (detects antibodies to the bacteria Treponema pallidum) Rapid Review Syphilis Primary: painless chancre Secondary: lymphadenopathy, comdyloma lata, rash on palms/soles Tertiary: gummas VDRL and RPR positive 4-6 weeks after infection Primary/secondary: IM benzathine penicillin G x 1 dose Tertirary: IM benzathine penicillin G x 3 weeks

Question: True or false: an oral cholera vaccine is recommended for travelers going to cholera-endemic regions?

Answer: False. Rapid Review Cholera Patient with a history of drinking contaminated water Complaining of severe, watery diarrhea flecked with mucus "rice water" PE will show dehydration Diagnosis is made by stool culture or darkfield microscopy Most commonly caused by Vibrio cholerae Treatment is supportive. Moderate to severe - fluoroquinolones, macrolides, and tetracyclines

Question: What are the clinical signs of subcutaneous heroin injection?

Answer: Fat necrosis, lipodystrophy, atrophy over the extremities, and skin abscesses.

Question: What type of heart block is consistent with acute rheumatic fever?

Answer: First degree heart block (prolonged PR interval). Rapid Review Rheumatic Fever Patient with a history of GAS infection Complaining of fever, red skin lesions on the trunk and proximal extremities, and small, non-tender lumps located over the joints PE will show JONES criteria: Joints, Oh, no carditis!, Nodules, Erythema marginatum, Sydenham's chorea Labs will show anti-streptolysin O, anti-DNase B, positive throat culture, or positive rapid antigen test Treatment is antibiotics, NSAIDs Comments: Modified Jones Criteria for a first episode of acute rheumatic fever: need 2 major or 1 major and 2 minor

Question: Which congenital disease of fetal skin causes ectropion?

Answer: Harlequin-type icthyosis. Rapid Review Ectropion Patient will be complaining of tearing and symptoms of dry eyes PE will show red conjunctiva and the eyelid turns outward Most commonly caused by aging

Question: What is the most common adverse effect of nitroglycerin ointment?

Answer: Headache. Rapid Review Anal Fissure Patient will be complaining of rectal pain and bleeding which occurs with or shortly after defecation PE will show fissure located in the posterior midline Diagnosis is made by visual inspection Treatment is stool softeners, protective ointments, sitz baths Comments: If fissures are located laterally, search for pathologic etiologies

Question: What kind of anemia is associated with Mycoplasma pneumoniae?

Answer: Hemolytic anemia secondary to cold agglutinins. Rapid Review Atypical Pneumonia Patient will be complaining of gradual onset of dry cough, dyspnea, and extra-pulmonary symptoms such as headache, myalgias, fatigue, and GI disturbance PE will show rales with auscultation of lung fields Most commonly caused by Mycoplasma pneumoniae Treatment is azithromycin

Question: What side effects are likely to occur in patients using intravesical chemotherapy?

Answer: Intravesical chemotherapy typically results in irritative voiding symptoms. Rapid Review Bladder Cancer Patient will be older With a history of smoking Complaining of painless hematuria Diagnosis is made by cystoscopy Most common type is transitional cell carcinoma

Question: What is a potential side effect of ordering a contrast enhanced CT while someone is taking metformin?

Answer: Lactic acidosis. Rapid Review Nephrolithiasis Patient will be complaining of flank pain radiating to groin PE will show a patient that won't lay still and hematuria Diagnosis is made by helical CT Most commonly caused by calcium oxalate Struvite: staghorn calculi, urease producing bacteria Uric acid: radiolUcent on xray, gout Cystine: children with metabolic diseases Most common location is the ureterovesiclular junction (UVJ) Treatment is: < 5 mm: likely to pass spontaneously > 8 mm: unlikely to pass, lithotripsy

Question: What is the most frequently occurring pelvic tumor in women?

Answer: Leiomyoma (uterine fibroid). Rapid Review Uterine Fibroids (Leiomyoma) Patient will be an African-American woman, 20 - 40 years old Complaining of menorrhagia and dysmenorrhea PE will show a palpable, asymmetric, non-tender uterus Diagnosis is made by pelvic ultrasound Majority do not require surgical or medical treatment Severe cases: Myomectomy (fertility can be preserved) or hysterectomy

Question: What dermatophyte causes tinea versicolor?

Answer: Malasezzia species such as M. globosa and M.furfur. Rapid Review Pityriasis Rosea Patient with a history of a larger lesion one week prior, "herald patch" Complaining of rash on the back PE will show diffuse papulosquamous rash on the trunk, "Christmas tree-like" distribution Treatment is self-limiting disease, antihistamines for itching

Question: What is the most common intracranial complication of acute otitis media?

Answer: Meningitis. Rapid Review Acute Otitis Media Patient will be an infant or young child Complaining of ear pain, fever, URI symptoms PE will show TM erythema and decreased mobility of TM Most commonly caused by viral > bacterial (S. pneumoniae most common) Treatment is amoxicillin

Question: At which level do most lumbar disc herniations occur?

Answer: More than 95% of disk herniations occur at L4 or L5.

Question: Renal agenesis, another congenital genitourinary anomaly, presents with which fetal symptoms?

Answer: Oligohydramnios, pulmonary hypoplasia and facial abnormalities. Fetal demise is common.

Question: What is the common side effect of rifampin?

Answer: Orange discoloration of body fluids. Rapid Review Pulmonary Tuberculosis (TB) RFs: immunodeficiency, immigrant, close contact Latent/primary TB: asymptomatic Active/reactivation TB: fever, night sweats, weight loss, productive cough, hemoptysis Erythema nodosum Primary TB CXR: Ghon focus Active/reactivation TB: upper lobes, cavitary lesions Dx: sputum smears for acid-fast bacilli (AFB), sputum/tissue culture for AFB (gold standard) PPD: gold standard for latent TB dx Latent rx: 6-9 months of INH Primary rx: Rifampin, INH, Pyrazinimde, Ethambutol (RIPE)

Question: What is the classic surgery for resection of a carcinoma of the head of the pancreas?

Answer: Pancreaticoduodenectomy, also known as a Whipple procedure. Rapid Review Pancreatic Cancer Patient with a history of smoking Complaining of painless jaundice, depression, and weight loss PE will show palpable nontender gallbladder (Courvoisier sign), migratory thrombophlebitis (Trousseau's syndrome) Labs will show CA 19-9 serum marker Most common type is adenocarcinoma Comments: Other PE signs: Palpable left supraclavicular lymph node (Virchow's node), palpable nodule bulging into the umbilicus (Sister Mary Joseph sign)

Question: What is Fitz-Hugh-Curtis?

Answer: Peri-hepatitis - a complication of pelvic inflammatory disease. Rapid Review Trichomoniasis Patient will be a woman complaining of malodorous vaginal discharge PE will show frothy, green/yellow discharge, "Strawberry cervix" Labs will show pH >5, flagellated, motile, pear shaped Diagnosis is made by wet mount Most commonly caused by Trichomonas vaginalis Treatment is metronidazole

Question: What other population is protected under the laws of elder abuse?

Answer: Persons with disabilities, often referred to as dependent adults. Rapid Review Elder Abuse and Neglect Patient disabled, demented Caregiver reluctant to leave patient alone with physician Unexplained injuries Patient appears fearful of caregiver Poor hygiene/malnourished Notify adult protective services

Question: What is the most common system involved in children with active tuberculosis?

Answer: Pulmonary.

Question: What is the most common type of retinal deattachment?

Answer: Rhegmatogenous retinal detachment. It generally occurs in patients older than 45 years and is associated with degenerative myopia. Rapid Review Central Retinal Vein Occlusion Varying degree of vision loss Monocular vision loss Fundoscopic exam: "blood and thunder" Ophthalmology consultation

Question: What is the formula to calculate serum osmolality?

Answer: Serum OSM = (2 [Na]) + ([glucose]/18) + (BUN/2.8) Rapid Review Hyponatremia Serum Na <135 Severity depends on rate of development of symptoms Nausea, vomiting, paresthesias, AMS Central pontine myelinolysis if corrected too fast Hypertonic saline for symptomatic patients

Question: What are some symptoms seen in early ethanol withdrawal?

Answer: Sweating, flushing, sleep disturbances, hallucinations, seizures, mild mental status changes. Rapid Review Substance Abuse Use → impairment or distress Not dependent

Question: What screening tool or questionnaire is most often used to screen for bipolar disorder?

Answer: The Mood Disorder Questionnaire (MDQ) is a tool that combines DSM-IV criteria and clinical experience to screen for bipolar disorder. Rapid Review Bipolar disorder Lifelong, extreme mood swings Mania, hypomania Major depressive disorder Inflated self esteem Decreased need for sleep Pressured speech Flight of ideas Excessive pleasurable activity

Question: What is the classic maneuver to test the integrity of the Achilles tendon?

Answer: The Thompson test: Patient is prone and knee at 90 degrees. Squeezing the calf muscle should cause plantar flexion of the foot with an intact Achilles tendon. Rapid Review Achilles Tendon Rupture Patient will be a deconditioned athlete With a history of fluoroquinolone use Complaining of "pop" or "snap" and sudden pain in the calf area PE will show absent plantarflexion upon calf squeeze (Thompson test) Treatment is posterior splint in plantarflexion, orthopedic consult

Question: What is the average fluid deficit in patients with hyperosmolar hyperglycemic state?

Answer: The average is 20% to 25% of total body water, or 8 to 12 L. Rapid Review Hyperosmolar Hyperglycemic State Hyperglycemia → dehydration w/o acidosis Type 2 DM AMS Glc > 600 Negative ketones Rx: IVF

A 35-year-old previously healthy man presents to your office with a complaint of diarrhea 24 hours after returning from a mission trip to Sierra Leone. He endorses hourly bouts of large volumes of stool that now have a white-colored liquid appearance. Physical exam findings include decreased skin turgor, dry buccal mucous membranes and a rapid, thready radial pulse. Which of the following is the most appropriate next step in management? Administer intravenous fluids Administer vitamin C Chloroquine Trimethoprim-sulfamethoxazole

Correct Answer ( A ) Explanation: Cholera is an acute diarrheal illness caused by the gram negative bacterium Vibrio cholera. It is found in regions with limited access to resources and clean water and is endemic in approximately fifty countries, mostly in Asia and Africa. Cases in the United States are seen in patients who have traveled to countries with cholera epidemics. Symptoms of cholera include severe diarrhea, abdominal pain, borborygmi, and vomiting. The classic description of the diarrhea seen in patients with cholera is "rice water stool", which refers to watery stool with flecks of mucus. Fever is generally not seen in patients with cholera, and an alternative etiology should be pursued in patients with fever and diarrhea. Diagnosis is by clinical suspicion and may be confirmed with stool culture. The mainstay of treatment for cholera is aggressive volume repletion given orally in mild cases and intravenously in patients with severe volume depletion or shock. Patients should be monitored to assess volume status and determine appropriate fluid repletion. Antibiotics are used as an adjunctive therapy and options include fluoroquinolones, macrolides and tetracyclines. Adequate nutrition is important in patients with severe diarrhea. Supplementation with vitamin A and zinc should be given in patients with cholera to address nutritional losses and help reduce the duration and volume of stool. Vitamin C (B) does not play a specific role in the treatment of cholera. Chloroquine (C) is used in the treatment of malaria. Most V. cholerae strains are resistant to trimethoprim-sulfamethoxazole (D).

A 71-year-old man is discovered to have gross hematuria during a routine physical exam. This is a new finding, and is confirmed by a repeat, clean-catch urinalysis. He denies any urinary urgency, frequency, or dysuria. No flank masses are palpated. Which of the following risk factors would have contributed to his most likely disease? Cigarette smoking Cryptorchism Heavy alcohol use HPV infection

Correct Answer ( A ) Explanation: Cigarette smoking is strongly related to the development of bladder carcinoma, of which this patient is showing symptoms. Bladder carcinoma typically presents in the 8th decade of life, most often in men. In addition to cigarettes, other risk factors include exposure to industrial dyes and solvents. The most common presentation is as painless gross or microscopic hematuria, as seen on a urinalysis. Rarely, urinary urgency, frequency, or dysuria may develop. After a urinalysis is performed, the next best step in diagnosing bladder cancer is a cystoscopy with biopsy. Abdominal ultrasounds, CT, or MRI may later be necessary for staging. Treatment depends on staging. More bladder cancers are superficial at diagnosis and can be managed with transurethral resection and intravesical chemotherapy. More invasive bladder carcinomas generally require radical cystectomy, irradiation, and combination chemotherapy regimens. Prognosis is usually good, with survival rates for superficial bladder cancers near 80%. Cryptorchism (B) is not a risk factor for bladder carcinoma. However, it is considered a significant risk factor for testicular cancer. Heavy alcohol use (C) is not an established risk factor for bladder cancer, though it is for several gastrointestinal malignancies. HPV infection (D) is not a risk factor for bladder cancer. It is more likely to contribute to the development of cervical cancer in women and anal cancer in both genders.

A 68-year-old woman with no significant medical history presents to the clinic with her daughter for cognitive changes. The daughter says that her mother gets "lost" in conversations, and that she would "sleep all day" if permitted. Further questioning reveals a recent history of constipation, as well as passing two kidney stones within the past 12 months. Which of the following electrolyte imbalances is most consistent with these symptoms? Hypercalcemia Hyperkalemia Hypocalcemia Hypokalemia

Correct Answer ( A ) Explanation: Hypercalcemia is associated with cognitive changes, malaise and fatigue, constipation, and renal calculi. Other symptoms may include polyuria, polydipsia, and bone pain. This can be remembered by the phrase "stones, bones, abdominal groans, psychic moans, and fatigued overtones." Hypercalcemia has many possible etiologies, though primary hyperparathyroidism remains the most common. Others include hypercalcemia of malignancy, multiple myeloma, sarcoidosis, prolonged immobilization, or excessive intake of calcium/vitamin D supplements. The most accurate measurement of serum calcium is the ionized calcium concentration and should be ordered whenever the total serum calcium abnormal. Until the primary cause is identified, hypercalcemia is initially managed with fluids and forced calciuresis. Intravenous saline is needed in severe cases. Intravenous bisphosphonates may be used in hypercalcemia that is due to hyperparathyroidism or a malignancy. Calcitonin can be given to manage hypercalcemia while the bisphosphonates reach therapeutic levels. Unmanaged hypercalcemia may lead to serious complications including neuromuscular manifestations such as weakness or paresthesia, or cardiac arrhythmias secondary to QT interval shortening or heart block. Hyperkalemia (B) is most commonly seen in patients with advanced kidney disease, though it may occur in patients taking ACE-inhibitors, angiotensin-receptor blockers, or potassium-sparing diuretics. Symptoms include muscle weakness, flaccid paralysis, and ileus. Hypocalcemia (C) causes increased excitation of nerve and muscle cells, leading to muscle cramps and tetany. The classic Chvostek sign (facial muscle contraction with tapping of the facial nerve) and Trousseau sign (carpal spasm with induced occlusion of the brachial artery) are seen. Hypokalemia (D) usually causes muscular weakness, fatigue, and muscle cramps in early stages, with ileus, arrhythmias, and rhabdomyolysis occurring as the imbalance progression.

A 17-year-old boy is brought to the emergency department by his friends. They were at a house party and found their friend unconscious after he ingested an unknown substance. On physical exam, vital signs are temp 96°F, heart rate 50, respiratory rate of 4, and oxygen saturation 92% on room air. The boy's pupils are miotic. On lung auscultation, there are crackles bilaterally. You administer supplemental oxygen. Which of the following is the most appropriate clinical intervention? Administer intravenous naloxone Begin intravenous fluids with sodium bicarbonate Initiate therapy with benzodiazepines Perform gastric lavage

Correct Answer ( A ) Explanation: Intravenous naloxone is an appropriate first step in managing a patient with acute heroin (opiate) overdose. Heroin is an addictive opiate that can be injected intravenously or subcutaneously, or can be snorted. It has a rapid onset of action, and easily crosses the blood brain barrier, producing euphoria. Other effects of the drug include decreased pain sensation, diminished level of consciousness, flushed skin, constipation, and pinpoint pupils. In an overdose, patients will exhibit hypothermia and respiratory depression, which can lead to coma and death. The withdrawal symptoms are reversed, including dilated pupils, restlessness, lacrimation, rhinorrhea, diarrhea, and sweating, as well as tachycardia and hypertension. Diagnosis of heroin intoxication is clinical and typically follows the opiate toxidrome of miosis, CNS depression, and hypoventilation. Naloxone is an opiate antagonist that will reverse respiratory depression and other effects caused by opiates. Other medications such as methadone and buprenorphine can assist with detoxification. Salicylate overdose (B) may cause respiratory depression, but should not affect the pupil size. Bicarbonate-containing fluids in salicylate toxicity can alkalinize the urine, which assists in renal excretion of the drug. Benzodiazepines (C) are used in alcohol withdrawal to prevent delirium tremens. Alcohol intoxication can result in altered mental status and respiratory depression, but should not cause miotic pupils. In a heroin overdose, benzodiazepines might worsen the respiratory depression, leading to intubation. In phencyclidine (PCP) overdose (D), the patient may exhibit altered mental status and aggressive behavior. The respiratory depression and miotic pupils would not be expected. Gastric lavage may be useful in treatment of PCP overdose, since PCP has poor gastric absorption. Gastric lavage would not be expected to improve symptoms in heroin overdose, especially if the heroin was administered intravenously or inhaled.

A patient with irritable bowel syndrome complains mainly of lower abdominal pain. She denies diarrhea or constipation, and rarely has problematic flatulence. For this patient's abdominal pain, which of the following medications do you recommend? Desipramine Loperamide Lubiprostone Rifaximin

Correct Answer ( A ) Explanation: Irritable bowel syndrome is considered an intestinal motility disorder in which chronic diarrhea is the main symptom. Associated symptoms include abdominal pain, bloating, flatulence and constipation. Some dysmotility causes include improper secretion due to luminal or environmental stimuli, decreased pain thresholds (hence, increased pain sensitivity) and dysregulation of the brain-intestinal axis. Rome III diagnostic criteria are based on symptom presence on >3 days per month over the past 3 months plus 2 or more of the following: defecation decreases symptoms, a change in the frequency of stooling or a change in the shape of stool. Treatment regimens are geared toward which of the symptoms a patient with irritable bowel syndrome suffers from most: pain, diarrhea, bloating or constipation. The pain associated with irritable bowel syndrome can be treated with selective-serotonin reuptake inhibitors like citalopram, tricyclic antidepressants like desipramine, and antispasmodics such as atropine, hyoscyamine, dicyclomine or scopolamine. Loperamide (B), a opioid piperidine-derivative, is used to treat diarrhea, not pain, associated with irritable bowel syndrome. Lubiprostone (C) is a prostaglandin E1 derivative. Its chloride channel agonism is used in treating the constipation, not pain, associated with irritable bowel syndrome. Bloating and flatulence that are associated with irritable bowel syndrome can be treated with rifaximin (D), a semisynthetic antibiotic.

Which of the following medications has the greatest impact on reducing mortality in patients presenting with acute coronary syndrome? Aspirin Beta blocker Morphine Nitroglycerine

Correct Answer ( A ) Explanation: Of the medications listed above, aspirin has been shown to have the greatest effect on reducing mortality in patients with acute coronary syndrome (ACS). Aspirin is an irreversible antiplatelet agent. It inactivates platelets for the life span of the platelet (8-10 days). ISIS-2 demonstrated a 23% reduction in mortality for patients with acute myocardial infarction (AMI) that received aspirin but not thrombolytic therapy. It also demonstrated that aspirin was synergistic with thrombolytics (42% reduction in mortality). Overall, aspirin has a number needed to treat (to prevent death) of 42. Administration of aspirin (chewed and swallowed) is recommended in any patient with suspected ACS. Beta-blockers (B) are used in STEMI to prevent ventricular dysrhythmias but have shown only a small (0.5 - 1%) improvement in this outcome. Morphine (C) is an opiate that has been used as an adjunct for relief of ischemic pain but has never been shown to improve mortality (and some studies have shown harm). Nitroglycerin (D) is a vasodilator that reduces anginal pain but has also never been shown to improve outcomes.

Which of the following structures of the ear contains auditory sensory receptors? Cochlea Semicircular canals Tensor tympani Vestibule

Correct Answer ( A ) Explanation: The ear includes the external, middle, and inner ear. The trumpet-shaped auricle of the external ear helps to funnel sound vibrations into the middle ear. The middle ear is an air-filled structure that includes the tympanic membrane and the auditory ossicles, which transmit vibration of the tympanic membrane to the inner ear. The inner ear is the most intracranial portion of the ear which processes sound and balance. Vibratory impulses from the auditory ossicles of the middle ear are transmitted into the fluid-filled cochlea. The cochlea contains tiny hair-like auditory receptors which detect vibratory input and convert them into electrical impulses which are transmitted to the brain via the auditory nerve. The inner ear also contains the vestibular labyrinth which processes balance. Cristae within the semicircular canals detect angular acceleration, and macule detect linear acceleration. The otolithic organs, the utricle and saccule, are housed within the vestibule, and process balance and orientation. The semicircular canals (B) contain cristae and macule which detect angular and linear acceleration, respectively. The tensor tympani (C) is a small muscle within the ear which arises from the auditory canal and connects to the malleus. The vestibule (D) contains the otolithic organs which process balance and orientation.

A 5-year-old boy with von Willebrand's disease presents with persistent bleeding after he lost a tooth 4 hours ago. Vital signs are unremarkable. Physical exam reveals oozing at the site of the tooth which persists despite pressure. Which of the following is the most appropriate treatment? Desmopressin Factor VIII concentrate Fresh frozen plasma Vitamin K

Correct Answer ( A ) Explanation: The patient has von Willebrand's disease, an autosomal dominant coagulation disorder in which von Willibrand's factor is low or absent. vWF facilitates platelet adhesion and links platelet membrane receptors to the endothelium. Mucosal bleeding is characteristic, such as seen in epistaxis, gingival bleeding, menorrhagia, and GI bleeding. Unlike hemophilia, hemarthrosis is rare. There are three types: 1) type I is the most common type with mild-to-moderate decrease in vWF, 2) type II with dysfunctional vWF, and 3) type III with no detectable vWF. Desmopressin (DDAVP) is the first-line treatment for bleeding and is sufficient in most patients. Factor VIII (B) is indicated for patients with severe disease (type II or III disease). Fresh frozen plasma (C) is a product from centrifuged whole blood which contains factors II, V, VII, IX, X, and XI. It should only be used in severe bleeding if factor VIII concentrate is not available. Vitamin K (D) is indicated for bleeding in patients taking vitamin K antagonists such as warfarin.

Which structure is compressed by a far lateral unilateral disc herniation between L4 and L5 lumbar vertebrae? L3 nerve root L4 nerve root L5 nerve root No nerve roots as this is below the level of the nerve roots

Correct Answer ( B ) Explanation: Far lateral unilateral disc herniation between L4 and L5 usually compresses the L4 root as it crosses the disc before exiting at the L4 intervertebral foramen. Pain may be localized around the medial side of the leg. Numbness may be present over the anteromedial aspect of the leg. The anterior tibial muscle may be weak, as evidenced by the inability to heel walk. The quadriceps and hip adductor group, both innervated from L2, L3, and L4, also may be weak and, in extended ruptures, atrophic. Reflex testing may reveal a diminished or absent patellar tendon reflex (L2, L3, and L4) or anterior tibial tendon reflex (L4). Sensory testing may show diminished sensibility over the L4 dermatome, the isolated portion of which is the medial leg and the autonomous zone of which is at the level of the medial malleolus. Unilateral disc herniation between L3 and L4 results in compression of the L3 (A) or L4 nerve root. A posterolateral unilateral disc herniation between L4 and L5 results in compression of the L5 root (C). The nerve roots are still present at this level (D). Although the spinal cord terminates near lumbar nerves L1 and L2, the spinal nerves continue as a bundle of nerves called the cauda equina.

A previously healthy 18-year-old woman presents with sore throat and pain with swallowing. Her vital signs are T 102.7°F, HR 124, BP 123/76, RR 22, and oxygen saturation 97%. On examination she has trismus, pain with neck extension, and difficulty swallowing her saliva. Her oropharyngeal examination is unremarkable. Which of the following is the most appropriate next step in management? CT scan of the neck with contrast and ENT consultation Ibuprofen, dexamethasone, and a Rapid strep test Oral antibiotics and ENT follow up Peritonsillar needle aspiration

Correct Answer ( A ) Explanation: This patient is suffering from a retropharyngeal abscess and will need advanced imaging (CT scan of the neck with IV contrast) to further delineate the extent of the disorder along with emergent ENT consultation for possible operative intervention. Historically, this was a disease of children under 6 years of age but adults are increasingly affected. A number of infectious processes including nasopharyngitis, otitis media, peritonsillar abscess, dental infections as well as iatrogenic procedures including endoscopy and dental instrumentation have been associated with retropharyngeal abscess formation. The infection is most commonly polymicrobial with both aerobes and anaerobes requiring broad antibiotic coverage. Patients typically present with sore throat, odynophagia, dysphagia, drooling, muffled voice, neck stiffness, fever and trismus. In severe cases, the patient may hold the neck in extension in order to increase airway diameter by distracting the posterior pharynx from the airway. CT scan and MRI are diagnostic but in unstable patients, lateral neck X-ray can demonstrate retropharyngeal swelling supporting the diagnosis. Additionally, if the patient is unable to lie flat for advanced imaging, direct visualization with an upper airway scope can be diagnostic. Oral antibiotics (C) alone are insufficient for treatment of retropharyngeal abscess in the majority of cases. In addition, the patient should not be discharged home as she is at risk to develop a compromised airway. Ibuprofen, dexamethasone, and a Rapid strep test (B) is the standard treatment for simple pharyngitis. However, this patient has a deep space infection and requires imaging, intravenous antibiotics, ENT consultation and possible surgery. In this patient, there is no swelling of the tonsils to suggest a peritonsillar abscess (D) as the cause of the patient's symptoms.

A 23-year-old woman presents with seizures. The patient received 2 mg of lorazepam by EMS but continues to seize. Serum lab tests show the following: sodium 118, potassium 3.6, chloride 90, bicarbonate 21, BUN/Cr 10/1.0, glucose 89. What treatment should be administered next? 3% hypertonic saline Dextrose Diazepam Pyridoxine

Correct Answer ( A ) Explanation: This patient presents with prolonged seizure activity and hyponatremia and should emergently be treated with hypertonic saline. Hyponatremia is defined as a serum sodium level <135 mEq/L and is the second most common electrolyte abnormality after hypokalemia. The symptoms and signs of hyponatremia depend on the patient's volume status, the cause and the rapidity of the change in serum sodium. Typically, patients with acute changes will have more severe symptoms including nausea, vomiting, confusion, stupor and seizures. Chronic hyponatremia will typically present with mild neurologic symptoms as well as lower serum sodium levels than acute hyponatremia. In patients without neurologic symptoms, volume status should be assessed and additional labs should be sent off to determine the cause of hyponatremia (urine sodium, osmolarity etc.). Patients with neurologic symptoms should be aggressively treated with 3% hypertonic saline. When correcting serum sodium, it is important to increase the serum sodium by no more than 0.5 mEq/L/hour and by no more than 10 - 12 mEq/day. More rapid changes can lead to central pontine myelinolysis, a crippling neurologic disease. Hypoglycemia is a common cause of seizures and should be immediately reversed with the administration of dextrose (B). Typically, patients are symptomatic with a blood sugar lower than 50-60 mg/dl. Diazepam (C) is a benzodiazepine that is frequently used in the treatment of seizures. However, it is not effective in hyponatremia. Pyridoxine (D) or vitamin B6 should be administered to patients with status epilepticus from isoniazid overdose.

Which of the following is most correct regarding treatment of Group A Strep pharyngitis? Treatment prevents acute rheumatic fever but not post-streptococcal glomerulonephritis Treatment prevents both acute rheumatic fever and post-streptococcal glomerulonephritis Treatment prevents colonization with Group A Strep in the future Treatment prevents post-streptococcal glomerulonephritis but not acute rheumatic fever

Correct Answer ( A ) Explanation: Treatment of Group A Strep pharyngitis prevents acute rheumatic fever, a non-suppurative, post-infectious complication of streptococcal pharyngitis. The diagnosis of acute rheumatic fever requires evidence of recent Group A Strep infection and the presence of either 2 major or 1 major and 2 minor Jones criteria. Acceptable evidence of recent Group A Strep infection includes a positive throat culture or rapid streptococcal antigen test or an elevated or rising anti-streptococcal antibody titer. Major Jones criteria include subcutaneous nodules, migratory polyarthritis, erythema marginatum, carditis, and chorea. Minor criteria include arthralgia, fever, elevated acute phase reactants (erythrocyte sedimentation rate or C-reactive protein), or a prolonged PR interval. Treatment of Group A Strep pharyngitis prevents acute rheumatic fever but does not prevent post-streptococcal glomerulonephritis (B, D). While appropriate antibiotics will eradicate Group A Strep from the oropharynx, treatment does not prevent colonization with Group A Strep (C) in the future. Currently, antibiotic therapy is not recommended to eradicate colonization with Group A Strep.

A young woman presents with a complaint of "stained underwear." She reports that for the last three days she has noticed a malodorous, greenish discharge emanating from her "groin." You take a thorough history and perform a pelvic examination. Which of the following is the next best step in evaluating this complaint? Bacterial culture Microscopic examination Pelvic ultrasonography Serum complete blood count and chemistries

Correct Answer ( B ) Explanation: Infective vaginitis is very likely given the above clinical description. In the initial evaluation of these symptoms, it is important to determine the causative agent. A KOH "whiff" test can be performed to detect the amine-like "fishy" odor of bacterial vaginosis or trichomonas vaginitis. However, direct microscopic examination of the discharge suspended in saline (termed wet preparation, or 'wet prep') will reliably establish the diagnosis, and subsequently direct proper therapy. Bacterial culture (A) has limited utility based on the fact that the vagina is normally colonized with several different bacteria. Ultrasound (C) will not determine which infectious agent is present. Basic laboratory tests (D) will also not reveal the causative agent. Plus, they would more than likely be normal in this stable patient.

A 19-year-old man presents with his parents who state that their son has shown signs of impulsivity, pressured speech, racing thoughts, and a decreased need for sleep over the last 3-4 weeks that has caused him to miss school. What is the most likely diagnosis? Attention-Deficit/Hyperactivity Disorder Bipolar disorder Major depressive disorder Schizophrenia

Correct Answer ( B ) Explanation: Bipolar disorder is a chronic mood disorder characterized by the presence of mania (bipolar I disorder) or hypomania and depression (bipolar II disorder). Manic episodes are distinct periods of abnormally and persistent moods that can be euphoric, expansive, or irritable. Manic patients often have greatly inflated self-esteem, confidence, decreased need for sleep, pressured speech, racing or crowded thoughts, distractibility, increased involvement in goal-directed activities (e.g., starting many projects but being unable to finish any), hypersexuality, and excessive involvement in pleasurable activities with a high potential for painful consequences. Despite mania being the defining characteristic of the disease, depressed moods tend to predominate, with bipolar I patients experiencing a 3:1 ratio of depression to mania over the course of the illness. Hypomania consists of manic episodes that do not lead to social or occupational dysfunction. Pharmacologic treatment for bipolar consists of mood stabilizers such as lithium, valproic acid, and carbamazepine. Bipolar is associated with a high rate of suicide (up to 15%) and several comorbidities such as substance abuse and anxiety disorders. Attention-Deficit/Hyperactivity Disorder (A) is associated with difficulty focusing and distractibility but not manic behavior. Patients with major depressive disorder (C) do not have manic or hypomanic episodes. The essential feature of a major depressive episode is a period lasting at least 2 weeks during which the patient experiences depressed mood or loss of interest or pleasure in almost all activities, a distinct change in usual self and clinically significant distress or changes in functioning. Schizophrenia (D) is not associated with mania and is characterized by hallucinations, delusions, behavioral disturbances, disrupted social functioning, and associated symptoms in what is usually an otherwise clear sensorium.

For the past two years, a 77-year-old man is more forgetful. He completely disagrees with this observation, but his daughter calls you concerned with her father's progressive memory loss. She states that her "stubborn" dad has become "more edgy" around the family, which only seems to occur at the end of the day. She adds that her dad has difficulties with remembering the names of the grandchildren. You schedule the patient and his daughter for an appointment. Which of the following tests will you most likely administer during this appointment? Confusion Assessment Method Mini-Mental State Examination PHQ-9 Stanford-Binet test

Correct Answer ( B ) Explanation: Elderly patients are at high risk of depression and cognitive changes, namely delirium (acute onset) and dementia (chronic onset). Dementia, an age-related progressive cognitive decline, affects 5% of those aged 71-80 years, and near 40% of those aged over 90 years. Many patients are unaware or ignore these cognitive changes, chalking them up to "getting old." Dementia presents with gradual, progressive memory loss. It is associated with word-finding and concentration problems, emotional lability, personality changes, social withdrawal, and difficulties with dressing, cooking, balancing the checkbook, and maintaining hygiene. There are multiple types of dementia: Alzheimer disease, vascular dementia, and other less common dementias. Alzheimer dementia is most common. Dementia can be evaluated with brief quantitative screening tests of cognitive function, such as the Folstein Mini-Mental State Examination (MMSE) or the Memory Impairment Screen. The MMSE may be useful to provide a baseline for future comparison. Controversy exists over the use of memantine and anticholinergic medications in the treatment of dementia. The Confusion Assessment Method (A) is used in evaluating delirium, not dementia. The Stanford-Binet test (D) is a cognitive ability and intelligence test used to diagnose developmental or cognitive deficiencies in children. It is not used in the evaluation of elder cognitive changes. The PHQ-9 (C) is used in the evaluation of depression, not dementia. Patients with dementia are less likely to self-report their cognitive problems than are patients with depression. Although this test could be given with the MMSE during the appointment, assessing for a decline in mood or self-care with further specific questioning would more appropriately be executed before a specific depression screen is administered. Furthermore, memory is usually intact, and symptoms are usually worse in the morning, with depression, as compared to dementia.

In an adult patient who is otherwise asymptomatic and identifies an engorged tick that has been attached for longer than 36 hours, what is the most appropriate medication for prophylaxis of Lyme disease? Clindamycin Doxycycline Penicillin Vancomycin

Correct Answer ( B ) Explanation: Lyme disease is transmitted by the bite of an Ixodes scapularis tick and is the most common vectorborne disease in the United States. Doxycycline is the drug of choice for prophylaxis against Lyme disease in an adult patient who is not pregnant and living in an endemic area. Nadelman and colleagues have reported that the administration of the medication within 72 hours of the time the tick was removed significantly reduces the likelihood of contracting Lyme disease. Clindamycin (A), penicillin (C), and vancomycin (D) have not been shown to be effective as prophylactic agents in Lyme disease.

A 12-year-old girl presents with a rash. Erythematous papules and plaques are noted over her back. A prominent, oval-shaped, scaling papule is present on her right upper back. What is the most likely diagnosis? Atopic dermatitis Pityriasis rosea Tinea corporis Tinea versicolor

Correct Answer ( B ) Explanation: Pityriasis rosea is a classic childhood exanthem. The rash begins with the appearance of a characteristic herald patch, an erythematous, oval-shaped, sharply demarcated lesion that is most common on the upper back, chest, or neck. The rash subsequently spreads down the trunk in a "Christmas-tree" distribution along the cleavage lines of the skin. In children, it may also affect the face, scalp, and extremities. The lesions may desquamate and result in post-inflammatory hyperpigmentation. Many children may be asymptomatic, while others may experience intense pruritus. The typical course of pityriasis rosea is self-resolution, which may take weeks to months. Some patients may require topical steroids to control itching. Atopic dermatitis (A), also known as eczema, is a common childhood disease. The rash of eczema consists of erythematous, pruritic plaques, which are often located over flexor surfaces. While atopic dermatitis is a consideration in this child, pityriasis rosea is a more likely diagnosis due to the classic herald patch and Christmas-tree distribution of papules. Tinea corporis (C) is a fungal dermatophyte infection. In immunocompetent hosts, it presents with annular or circular plaques that expand centrifugally. As the lesions expand, they develop slightly raised leading edges with whitening of the center. Single or multiple lesions may be present. In the above patient, the papules in Christmas-tree distribution along with the preceding herald patch, make pityriasis rosea more likely than tinea corporis. However, if the diagnosis is not clear, a KOH stain can be used to evaluate for the fungal hyphae of tinea corporis. Tinea versicolor (D) is a cutaneous fungal infection that is most common among adolescents. The cutaneous infection results in hyper-pigmented or hypo-pigmented papules, patches, and plaques. Scaling may also be present. A herald patch and Christmas-tree distribution are not present in tinea versicolor.

A 76-year-old man presents complaining of generalized weakness for the past two weeks. His wife states that he has almost no appetite and he spends all day lying in bed. Physical exam reveals a mildly emaciated male who responds slowly to questions. His skin is pale and he has hyperpigmented palmar creases. Laboratory workup reveals a serum sodium level of 130 mmol/L and a serum potassium level of 5.8 mmol/L. Which of the following laboratory values is consistent with this patient's presentation? Decreased antidiuretic hormone level Decreased cortisol level Decreased thyroid-stimulating hormone level Decreased thyroxine level

Correct Answer ( B ) Explanation: Primary adrenal insufficiency, also known as Addison's disease, results from the inability of the adrenal cortex to produce sufficient glucocorticoids and mineralocorticoids. Most common signs and symptoms of Addison's disease include anorexia, generalized weakness, skin hyperpigmentation, and hypotension. Hyponatremia and hyperkalemia result from a decrease in aldosterone production, which decreases the number of sodium-potassium antiporters in present the renal tubules. This leads to increased excretion of sodium and retention of potassium. Further workup would reveal a decreased cortisol level. Treatment consists of glucocorticoid and mineralocorticoid replacement. A decreased antidiuretic hormone (ADH) level (A) is seen in central diabetes insipidus. It is characterized by polydipsia, polyuria, and an increased serum sodium level. A decreased thyroid-stimulating hormone (TSH) level (C) is seen in hyperthyroidism, and is the result of negative feedback from overproduction of thyroxine, or thyroid hormone. Symptoms of hyperthyroidism include heat intolerance, weight loss, anxiety, tachycardia, and hair loss. Electrolyte abnormalities are not consistent with hyperthyroidism. A decreased thyroxine (T4) level (D) is characteristic of hypothyroidism. Symptoms of hypothyroidism include cold intolerance, weight gain, generalized weakness, and edema. Hypothyroidism does not cause electrolyte abnormalities.

Which of the following can cause a false positive rapid plasma reagin (RPR)? Aspirin use Autoimmune disease Owning a pet cat Young age

Correct Answer ( B ) Explanation: Rapid plasma reagin (RPR) is a diagnostic test that looks for non-specific antibodies to Treponema pallidum, the organism that causes syphilis. The RPR test has a high sensitivity but low specificity. False positives are seen in individuals with autoimmune disease, viral infections (EBV, hepatitis, varicella, measles), lymphoma, malaria, connective tissue disease, pregnancy, older age, and IV drug abuse. As a result of the low specificity, a positive RPR should always be followed up by a more specific treponemal test such as Treponema pallidum hemagglutination assay (TPHA) and Fluorescent Treponemal Antibody Absorption (FTA-ABS). The Venereal Disease Research Laboratory (VDRL) test is also sometimes used as a screening test but the RPR test is generally preferred due to its ease of use. Aspirin use (A) does not cause increased RPR false positives. Owning a pet cat (C) may expose patients to toxoplasma. However, healthy individuals typically remain asymptomatic, but will develop IgG antibodies to toxoplasma. Old age, not young age (D) may lead to increased RPR false positives.

A 19-year-old man presents with a painless ulcer on the shaft of his penis present for the last 3 days. Examination is also notable for bilateral non-tender inguinal adenopathy. He reports he is sexually active with men. What is the most appropriate course of action? Ceftriaxone IM Penicillin IM Rapid plasma reagen test Viral culture

Correct Answer ( B ) Explanation: Syphilis is caused by the spirochete Treponema pallidum. Transmission occurs during exposure of moist skin to an infected area as the organism does not survive on dry surfaces. Syphilis progresses through three stages of illness. Primary syphilis is characterized by the chancre shown in the picture above. The lesion occurs at the site of exposure and begins as an erythematous papule that ultimately ulcerates. The ulcer has raised edges, sharply demarcated borders and a clean base. Left untreated, the chancre resolves after 2-6 weeks. Bilateral painless inguinal adenopathy is sometimes present. Secondary syphilis develops 5-8 weeks after resolution of the chancre. Most commonly, this stage involves a diffuse total body rash beginning on the trunk and spreading distally, often involving the palms and soles. During this stage, condylomata lata may develop (broad-based papules in the perineal area). Syphilis then enters a latent period, which can last for years in immunocompetent patients and then manifest most commonly affecting the cardiac or nervous systems. The treatment of primary syphilis is penicillin G benzathine 2.4 million units IM. Ceftriaxone IM (A) is the treatment of choice for gonococcal infection. Unless gonococcal disease is disseminated, a rash is not present. Patients typically present with urethritis or cervicitis. The rapid plasma reagen test (C) is one of the non-treponemal serologic tests available for syphilis in addition to the Venereal Disease Research Laboratory (VDRL) test. Both of these non-treponemal tests become positive approximately two weeks after the appearance of the chancre and will likely be negative in the patient presented above. A viral culture (D) is an appropriate test if herpes simplex virus is suspected although these ulcerations are often painful with tender adenopathy.

A 10-year-old child presents for a well child check. He is currently living in a shelter with his family. A tuberculin skin test is placed subcutaneously. Which of the following meets the minimum criteria for a positive test in 48 hours? 10 mm diameter of erythema 10 mm diameter of induration 15 mm diameter of induration 5 mm diameter of induration

Correct Answer ( B ) Explanation: The above child has recently been living in a shelter environment. Exposure to a homeless population is a risk factor for exposure to tuberculosis. Other risk factors for exposure include birth in high-prevalence regions of the world or travel to high-prevalence regions of the world. Increased risk of tuberculosis exposure also occurs through exposure to adults with HIV infection, users of illicit drugs, residents of nursing homes, incarcerated persons, or institutionalized persons. Children who are at increased exposure risk should undergo tuberculin skin testing with follow up injection-site inspection at 48-72 hours. A positive test in a child with known tuberculosis risk factors is 10 mm diameter of induration or greater. Notably, only the diameter of induration can be used for measurement of the skin reaction; erythema alone cannot be included in measurement. 5 mm diameter of induration (D) is considered a positive skin test in children with a known tuberculosis exposure, with suspected pulmonary tuberculosis on chest imaging, with clinical evidence of tuberculosis disease, and in immunosuppressed patients. 15 mm diameter of induration (C) is the positivity threshold for children older than 4 years of age without any identified tuberculosis risk factors. Finally, 10 mm diameter of erythema (A) is not considered a positive skin test, as induration, not erythema, is used for accurate measurement.

A 34-year-old woman presents with pain behind her left eye that is worse with eye movements. She adds that the vision in that eye, which was formerly "perfect," has been getting worse over the past few days. An exam is notable for a visual acuity of 20/80 in her left eye, impaired color vision, and an afferent pupillary defect. Which of the following underlying conditions will this patient most likely have? HIV infection Hyperlipidemia Multiple sclerosis Polymyalgia rheumatic

Correct Answer ( C ) Explanation: Multiple sclerosis is strongly associated with optic neuritis, the most likely diagnosis in this patient. Optic neuritis, a demyelinating disease of the optic nerve, usually presents in female patients and causes unilateral loss vision loss to develop over several days. The degree of impairment can range from mild to no perception of light. Central visual field loss and impaired color vision may also occur. Patients often have pain behind the affected eye that worsens with ocular movements. A relative afferent pupillary defect may be seen in the affected eye and a fundoscopic exam may show swelling of the optic disk and flame-shaped peripapillary hemorrhages. The vision loss in acute optic neuritis usually resolves more rapidly when treated with intravenous methylprednisolone for 3 days, followed by an oral steroid taper. If vision loss and pain persist beyond 2 weeks, a CT or MRI scan of the head and orbits is necessary to rule out a lesion compressing the optic nerve. However, most cases of optic neuritis are related to multiple sclerosis, which is diagnosed in 50% of patients within 15 years of their first episode. Female gender and the presence of white matter lesions on a brain MRI make developing multiple sclerosis more likely. Starting interferon therapy in a patient with optic neuritis who has multiple white matter lesions can prevent development of multiple sclerosis in a percentage of patients. HIV infection (A) is not commonly associated with optic neuritis. It is more likely to cause HIV retinopathy which manifests as cotton-wool spots, retinal hemorrhages, and microaneurysms on a fundoscopic exam. Hyperlipidemia (B) is a strong risk factor for central and branch retinal artery occlusions. However, unlike optic neuritis, these normally cause sudden, painless, monocular vision loss. Polymyalgia rheumatic (D), a rheumatologic condition hallmarked by pain and stiffness in the shoulder and pelvic girdles, is strongly associated with temporal, or giant cell, arteritis. This causes visual abnormalities along with persistent headache and jaw claudication, which this patient did not have.

A 12-year-old previously healthy adolescent presents to clinic with one week of cough and fatigue. He has also developed intermittent fevers with a Tmax of 101°F. His cough and fatigue have been progressively worsening. Breath sounds are coarse throughout all lung fields. What is the treatment of choice? Amoxicillin Azithromycin Cefdinir Oseltamivir

Correct Answer ( B ) Explanation: The most likely etiology of pneumonia in adolescents are atypical bacteria such as Mycoplasma pneumoniae and Chlamydia pneumoniae. Atypical bacterial pneumonia, also known as "walking pneumonia," has a more gradual onset and indolent course than typical bacterial pathogens such as Streptococcus pneumoniae. Symptoms of atypical bacterial pneumonia often begin with generalized malaise and progress to cough and fever. Breath sounds may be diffusely coarse and include scattered crackles or wheezes, but focal findings are typically not present. Chest film is likely to reveal perihilar and interstitial infiltrates. The treatment of choice for atypical bacterial pneumonia is five days of azithromycin. Tetracyclines may also be used in rare cases of azithromycin allergy. Amoxicillin (A) is the drug of choice for typical bacterial pneumonia in children ages 6 months to 5 years due to its excellent coverage of pneumococcus and the lower incidence of atypical pathogens in this age group. High-dose amoxicillin is recommended to overcome possible bacterial resistance, although pneumococcal resistance has decreased with routine pneumococcal vaccination. Cefdinir (C) is a third generation cephalosporin that also can be used to treat typical bacterial pneumonia. It would be indicated for children with non-type-1 hypersensitivity reactions to penicillin. Oseltamivir (D) can shorten the duration of influenza infection if given within the first 48 hours of symptom onset. It is recommended for children at high risk of developing complications of influenza, such as children < 2 years old, children with chronic conditions, and children on aspirin therapy. However, this patient's somewhat indolent and prolonged course makes influenza less likely than atypical pneumonia. Influenza pneumonia has a rapid and severe onset. In addition, the patient would not qualify for oseltamivir after one week of symptoms.

Which of the following is the first line medical treatment for symptomatic hypercalcemia? Bisphosphonates Calcitonin Fluid rehydration Furosemide

Correct Answer ( C ) Explanation: Aggressive rehydration with intravenous fluids is the initial treatment for patients diagnosed with hypercalcemia. Patients with calcium levels greater than 14 mg/dL or symptomatic patients with calcium levels greater than 12 mg/dL should be immediately and aggressively treated with fluid resuscitation. Normal saline should be used to achieve a urine output of 200 mL/hour. Volume depletion leads to enhanced renal sodium loss, which tends to perpetuate the hypercalcemia by increasing sodium resorption in the kidneys. Common manifestations of hypercalcemia include nephrolithiasis, bone pain, QT interval shortening, gastrointestinal manifestations and psychiatric effects - remembered by the mnemonic Stones, Groans, Moans, and Psychiatric overtones). Primary hyperparathyroidism and malignancy account for more than 90% of hypercalcemia cases. Other causes include medications such as lithium and thiazide diuretics, Vitamin-D intoxication, granulomatous disease, immobilization, and other endocrine disorders. When evaluating for hyperparathyroidism, it is important to obtain a full history, exam and medication history as well as an intact parathyroid hormone (PTH) level. Furosemide (D) is often used once adequate volume is restored. Furosemide inhibits sodium and chloride reabsorption in the kidney, therefore limiting hypercalcemia resorbtion. Calcitonin (B) and bisphosphonates (A) are both effective at lowering calcium levels by inhibiting osteoclastic bone resorption and are both used for hypercalcemia once hydration has been initiated. Bisphosphonates are only used in hypercalcemia of malignancy and will take 72 hours to act, whereas calcitonin is used in all causes of hypercalcemia and is rapid acting, however tachyphylaxis after 24 hours is a common side effect. In resistant, life-threatening hypercalcemia, hemodialysis should be instituted.

Which one of the following is the most common atypical presenting complaint in an elderly patient diagnosed with acute coronary syndrome? Diaphoresis Dizziness Dyspnea Weakness

Correct Answer ( C ) Explanation: Atypical features of acute coronary syndrome are present with increasing frequency in sequentially older populations. Elderly patients commonly present with atypical features of acute ischemic heart disease. In fact, atypical features are even more common than chest pain, with 60%-70% of patients older than 85 presenting with an angina equivalent complaint—most commonly dyspnea. Diaphoresis (A), dizziness (B), and weakness (D) are all atypical features of acute coronary syndrome but occur less commonly than dyspnea does.

Computed tomography imaging for which one of the following conditions is best done without contrast? Acute appendicitis Diverticulitis Nephrolithiasis Pulmonary embolism

Correct Answer ( C ) Explanation: Disease processes that involve calcifications may benefit from noncontrast-enhanced images because contrast may mask the appearance of calcifications. Typically, computed tomography, focusing on vascular, renal or pancreatic pathology should include noncontrast-enhanced images to identify calcifications. Common indications for computed tomography without contrast include suspected stroke within the first 3 hours of symptom onset, closed head injury, diffuse lung disease, soft-tissue swelling, infection or trauma of the extremities, suspected kidney stone, and suspected spinal trauma. Contrast agents are used to highlight blood vessels and to enhance the structure of organs like the brain, spine, liver, and kidney. Evaluation using contrast-enhanced computed tomography is indicated in the following common scenarios: acute appendicitis (A), cancer staging, diverticulitis (B), suspected complications of inflammatory bowel disease, pancreatitis, and suspected pulmonary embolism (D).

A 79-year-old woman presents with a dry, painful right eye. During inspection, you notice her lower lid has turned outward on itself, pulling away from the eyeball. Slit lamp examination shows no abnormalities. The conjunctiva do not appear injected, and the iris has no erythema. Which of the following is the most likely diagnosis? Anterior uveitis Corneal abrasion Ectropion Keratitis

Correct Answer ( C ) Explanation: Ectropion, the outward turning of the lower eyelid, is mainly due to the aging process. Weakening of the periorbital connective tissue allows the eyelid to fall away from the eyeball, leading to dry painful eyes. Associated symptoms include excessive tearing, blepharitis and non-infectious conjunctivitis. Other causes include scarring from burn injuries, congenital disease such as Down syndrome, and cranial nerve 7 palsies. Artificial tears may be of help, but if not, then treatment is surgical. Left untreated, complications of infection, corneal abrasion or corneal ulcer may arise. As a general rule, any red eye with pain should prompt an ophthalmology consultation. Anterior uveitis (A), also known as iritis or iridocyclitis, is inflammation of the iris (pigmented fibrovascular tissue and epithelium) and anterior chamber (posterior to cornea, contains aqueous humour). Corneal abrasion (B) refers to a defect in the corneal epithelium, typically from some mechanical injury. Slight lamp examination with fluorescein stain would typically illuminate an abrasion of the cornea. Keratitis (D) is the term used for generalized corneal inflammation, which is not present in the above patient.

A 19-year-old man with a BMI of 35, comes to your office with polyuria, polydipsia and polyphagia. In the process of working him up you determine that he may have diabetes. Which of the following tests can you use to diagnose him with diabetes? Any random glucose greater than 175 C peptide level Fasting plasma glucose greater than 126 on 2 occasions Glucose greater than 175 two hours after a 75 gram glucose load

Correct Answer ( C ) Explanation: Fasting serum blood glucose > 126 mg/dL on two separate occasions meets the ADA diagnostic criteria for diabetes. The other criteria include: (1) symptoms of hyperglycemia, such as weight loss, polyuria, and polydipsia, associated with a random serum glucose concentration >200 mg/dL; (2) A plasma glucose of >200 mg/dL two hours after a 75 gram glucose load during an oral glucose tolerance test. Fasting serum glucose concentration of 100 to 125 mg/dL are considered to be prediabetes. These individuals have an increased risk of progressing to diabetes by various pathophysiologic processes. Any random glucose >175 (A) is not criteria to diagnose diabetes. A glucose >200, not 175 (D) two hours after a 75 gram glucose load is diagnostic of diabetes. C peptide (B) is used to help distinguish between type 1 and type 2 diabetes and is not used as diagnostic criteria in diabetes.

Which of the following is a risk factor for an Achilles tendon rupture? Age older than 60 years Diabetes Fluoroquinolone usage Steroid taper

Correct Answer ( C ) Explanation: Rupture of the Achilles tendon is relatively common. It is most common in men between the ages 30 and 50 years who participate in recreational sports—the weekend warrior. However, it can occur in serious athletes. Exogenous risk factors include chronic corticosteroid usage and fluoroquinolone usage. Age older than 60 years (A) is not a risk factor. Most cases occur in patients between the ages of 30 and 50 years. Diabetes (B) is not a risk factor for Achilles tendon rupture. Chronic, rather than short-term, steroid usage (D) predisposes to Achilles tendon rupture.

A 53-year-old man presents with weight loss, anorexia, abdominal pain, jaundice and early satiety. Which of the following is the most appropriate initial diagnostic test to diagnose and stage pancreatic cancer? Biopsy CEA 19-9 antigen CT scan MRI

Correct Answer ( C ) Explanation: For patients presenting with common symptoms of pancreatic cancer, pancreas protocol CT is the standard for diagnosis and staging. Pancreas protocol CT involves triphasic (arterial, late and venous phases) cross-sectional imaging that allows for enhancement between the parenchyma and adenocarcinoma. Moderate to high risk factors for pancreatic cancer include: family history of pancreatic cancer, hereditary pancreatitis, familial atypical multiple mole melanoma, Peutz-Jueger syndrome, cystic fibrosis and the BRCA-2 gene carrier. Routine screening is not recommended for asymptomatic adults who are at average risk. However, it may be reasonable to consider screening persons at high risk of developing pancreatic ductal adenocarcinoma, such as persons from families with known genetic defects predisposing them to pancreatic cancer. If a pancreatic mass on a CT is identified, subsequent ultrasonography and fine-needle aspiration are indicated. CEA19-9 (B) is the most common serum tumor marker used to identify pancreatic ductal adenocarcinoma. This marker is useful to confirm the diagnosis and predict prognosis and recurrence after resection. Cancer antigen 19-9 is not tumor-specific; therefore it is not the gold standard for screening as it has a limited sensitivity of 50-75% and specificity of 80-85%. This marker is used to monitor for recurrence once the tumor has been resected. Once a mass is identified on CT, fine needle biopsy (A) is the next step to confirm the diagnosis. If CT is not possible because of an allergy to contrast media or renal failure then magnetic resonance imaging (MRI) (D), as well as magnetic cholangiopancreatography, can be performed as an adjunct to CT in detecting extrapancreatic disease.

Which of the following is true regarding hyperosmolar hyperglycemic state? Cerebral edema is a common complication due to rapid hydration Elevated serum potassium is commonly seen It is associated with a significantly higher mortality than diabetic ketoacidosis It is caused by increased insulin sensitivity

Correct Answer ( C ) Explanation: Hyperosmolar hyperglycemic state is defined by progressive hyperglycemia and hyperosmolarity that generally occurs in debilitated patients who have poor access to water; poorly controlled or undiagnosed type II diabetes; and commonly, a precipitating medical event. The condition is characterized by severe hyperglycemia with serum glucose usually >600 mg/dL, an elevated calculated plasma osmolality of >315 mOsm/kg, serum bicarbonate >15 mEq/L, an arterial pH >7.3, and serum ketones that are negative to mildly positive. Because it is associated with concurrent illness, the hyperosmolar hyperglycemic state has a much higher mortality rate than diabetic ketoacidosis has. Cerebral edema (A) is uncommon in hyperosmolar hyperglycemic state (unlike DKA, particularly in children). Potassium (B) losses are usually significant. Although there is a total body potassium deficit, in cases of severe acidemia, the serum level of potassium may be elevated. This is less common than in DKA. Hyperosmolar hyperglycemic state is caused, in part, by insulin resistance, deficiency, or both (C).

A 62-year-old woman presents with loss of urine when she coughs or sneezes. What type of incontinence is this? Functional Overflow Stress Urge

Correct Answer ( C ) Explanation: Incontinence is characterized by the unintentional loss of urine. Causes are multifactorial. Stress incontinence is an unintentional loss of urine during a stress maneuver (sneezing, coughing, laughing). It is due to weak pelvic floor or insufficient internal uretheral sphincter strength. It most commonly occurs in menopausal women who have had previous vaginal deliveries. It also occurs in men following prostate surgery. Nocturnal symptoms are uncommon. Treatment includes scheduled voiding, Kegel exercises to strengthen pelvic floor muscles, topical estrogen, alpha agonists, periurethral injections and surgery (highest cure rate). Functional incontinence (A) is due to functional or cognitive impairment resulting in an inability to use a toilet. Overflow incontinence (B) is the leakage of urine due to an over distended bladder resulting from detrusor weakness or mechanical forces including bladder outlet obstruction or an enlarged prostate. Medications including alpha blockers and anticholinergics can also lead to overflow incontinence. Symptoms include dribbling, frequency, hesitancy, nocturia and a weak stream. Urge incontinence (D), also known as detrusor hyperactivity, is the most common type of incontinence in the ambulatory elderly population. It is the leakage of urine secondary to an inability to delay urination after a full bladder is sensed. Symptoms include nocturia and frequency. It is caused by age-related smooth muscle changes, central nervous system dysfunction and sensory or motor bladder deficits.

Which of the following is true regarding uterine fibroids? Increase in size during menopause More common in White women than African American women Surgical removal is associated with a 25% to 30% rate of recurrence Typically occur as a single fibroid

Correct Answer ( C ) Explanation: Leiomyomas (uterine fibroids) are benign tumors of muscle cell origin that cause pain and abnormal bleeding. Uterine fibroids are associated with severe pain when part of the fibroid undergoes torsion or degeneration (due to rapid growth and loss of blood supply most common in early pregnancy). Diagnosis is made by ultrasound. The treatment depends on size and symptoms. Initial management usually includes NSAIDs, medroxyprogesterone, and gonadotropin-releasing hormone agonists. Surgical removal is associated with a 25% to 30% rate of recurrence and significant bleeding complications. Uterine fibroids decrease in size during menopause (A) and enlarge early in pregnancy. Uterine fibroids are 2x more common in African American women (B) than in White women. Uterine fibroids are usually multiple in nature (D) rather than single.

Which of the following is true regarding active tuberculosis? A cavitary lesion on CT of the chest is pathognomonic Isoniazid treatment for six months is adequate therapy It may have any appearance on chest X-ray Patients with active tuberculosis need droplet precautions

Correct Answer ( C ) Explanation: Most patients evaluated for tuberculosis present with reactivation of an old infection. When an immunocompetent person is exposed to tuberculosis, the immune system effectively gains control over the infection in the lung. It remains quiet and often never re-activates during a person's lifetime. Most patients are asymptomatic during primary infection and only develop symptoms during a re-activation. Approximately 8-10% of persons who do not take chemoprophylaxis after a primary infection (typically identified through a positive skin PPD test) will develop active tuberculosis. Cough is the most common symptom of pulmonary tuberculosis. Additionally, patients may develop fever (more common in the afternoon or evening), night sweats and hemoptysis. Due to the effects of cytokines (particularly tumor necrosis factor alpha), patient often lose weight. The classic X-ray finding on chest radiograph is a cavitary lesion in the upper lobe of the lung. However, tuberculosis can cause any abnormality on the X-ray including infiltrate in any portion of the lung. Lymphadenopathy is commonly seen in the hilum on X-ray. A cavitary lesion on CT of the chest (A) is not pathognomonic for tuberculosis. There are multiple other infectious causes of cavitary lesions in the chest including Staph aureus, Klebsiella pneumoniae, Pseudomonas aeruginosa, and fungal lesions. Isoniazid treatment for six months (B) is the recommended treatment for someone with evidence of a primary infection identified through a PPD test. For active tuberculosis, patients receive multiple anti-tuberculosis agents to which the organism is susceptible. Worldwide, increasing rates of drug resistant tuberculosis has been seen. The most common drug regimen is remembered by the mnemonic RIPE standing for rifampin, isoniazid, pyrazinamide, and ethambutol. Patients with active tuberculosis need airborne precautions, not droplet precautions (D). The size of the droplets is quite small necessitating airborne precautions in a negative pressure room with recirculating air.

A previously healthy 35-year-old woman presents to your clinic with questions about breast cancer screening. She has no family history of breast cancer and wants to know when she should start screening. Per the United States Preventive Services Task Force, which of the following is the most appropriate next step in management? Initiate screening magnetic resonance imaging at age 40 Initiate screening mammography at age 40 Initiate screening mammography at age 50 Initiate screening ultrasound at age 50

Correct Answer ( C ) Explanation: Screening guidelines for cancer are developed after consideration of a number of factors including the benefits and harms of screening, scientific evidence of the risks of the condition and cost. Guidelines for breast cancer screening have more scientific evidence than any other type of cancer screening and have changed in recent years. There remain some discrepancies between different medical groups regarding recommendations for screening. The United States Preventive Services Task Force recommends screening mammography for women every two years from ages 50-74 years. Earlier and more frequent screenings may be individualized based on patient risk factors such as family history of breast cancer. Screening mammography for women may start at age 40 (B) in individualized cases based on risk factors. Magnetic resonance imaging (A) is not recommended as a diagnostic tool in breast cancer screening since there is not enough evidence to support its use. Ultrasound (D) is not used as a screening tool in breast cancer screening. Women who have suspicious lesions on mammography are generally sent for ultrasound to better visualize the lesions and determine next steps.

Which of the following is included in the criteria of substance abuse? Great deal of time spent in activities to obtain, use, or recover substances Persistent desire or unsuccessful efforts to cut down or control use Persistent or recurrent social or interpersonal problems caused or exacerbated by substance Substance taken in larger amount or for longer period than intended

Correct Answer ( C ) Explanation: Substance abuse is the compulsion to use substances despite adverse consequences. One or more of the following must be seen in a 12-month period: continued use despite persistent or recurrent social or interpersonal problems caused or exacerbated by substance, recurrent substance related legal problems, recurrent use in physically hazardous situations, recurrent use resulting in failure to fulfill major role obligation at work, home, or school. These symptoms must never have met dependence criteria for the same class. Substance dependence is the physiologic or emotional reliance on a substance. Substance dependence includes three or more of the following in a 12-month period: great deal of time spent in activities to obtain, use, or recover (A), persistent desire or unsuccessful efforts to cut down or control use (B); substance taken in larger amount or for longer period than intended (D); tolerance (needing more for same effect, or diminished effect with same amount), characteristic withdrawal symptoms, important social, occupational, or recreational activities given up or reduced, and use continues despite knowledge of having a physical or psychological problem worsened by the substance. NOTE The psychiatric diagnoses, substance abuse and substance dependence, in DSM-IV-TR were replaced by one diagnosis, substance use disorder, in DSM-5. Although the crosswalk between DSM-IV and DSM-5 disorders is imprecise, substance dependence is approximately comparable to substance use disorder, moderate to severe subtype, while substance abuse is similar to the mild subtype. DSM-5 criteria for SUD are consistent across substances.

Which of the following diseases presents with small grouped vesicles on an erythematous base? Aphthous ulcer Dyshidrotic eczema Herpes simplex Impetigo

Correct Answer ( C ) Explanation: The herpes simplex viruses comprise 2 distinct types of DNA viruses: herpes simplex virus (HSV)-1 and HSV-2. HSV-1 causes oral lesions in approximately 80% of cases and genital lesions in 20% of cases. The reverse is true for HSV-2, which causes genital lesions in 80% of cases and oral lesions in 20% of cases. Herpetic lesions are classically described as small grouped vesicles on an erythematous base (dew drop on a rose petal). Aphthous ulcer (A) is a common noncontagious mouth ulcer also referred to as a canker sore. The cause is not completely understood but may involve a T cell-mediated immune response triggered by a variety of factors. These are ulcers and do not contain vesicles. Dyshidrotic eczema (B) is a vesicular rash typically found on the palms, soles, and sides of fingers that presents in the 3rd decade of life with lifelong occurrences. Although it has the similar vesicular appearance as herpes, the lesions of dyshidrotic eczema are opaque and deep-seated; they are either flush with the skin or slightly elevated and do not break easily. Eventually, small blisters come together and form large blisters and generally do not occur on an erythematous base. Impetigo (D) is characterized by honey-colored crusting lesions caused by staphylococci or streptococci most commonly on the face or other exposed areas.

Which of the following is associated with central retinal vein occlusion? Complete vision loss Flashing lights and floaters Retinal hemorrhages and disk edema on fundoscopy Vitreous hemorrhage

Correct Answer ( C ) Explanation: There are 2 types of central vein occlusion, nonischemic and ischemic. Nonischemic occlusion involves mild fundus changes and is not associated with an afferent pupillary defect (APD). These patients tend to have less severe vision loss. Ischemic occlusion is associated with worse visual acuity and often an afferent pupillary defect. On fundoscopy, the appearance can vary but classically includes dilated and tortuous veins, retinal hemorrhages, and disk edema. Hemorrhages can cover the entire fundus, giving a "blood and thunder" appearance. Treatment is complex and includes lowering intraocular pressure, applying topical steroids, cyclocryotherapy, and photocoagulation. The prognosis depends on the degree of obstruction and resultant complications. Vision loss accompanying central retinal vein occlusion can vary, ranging from minimal vision loss to recognition of hand movements only (A). Patients with retinal breaks and detachments complain of flashes of light related to the traction of the retina, floaters related to vitreal blood or pigmented debris (B), and visual loss. The visual loss is often described as a filmy, cloudy, or curtain-like appearance. Vitreous hemorrhage (D) results from bleeding into the pre-retinal space or into the vitreous cavity. The most common causes are diabetic retinopathy and retinal tears.

A 48-year-old man presents with a 4-week history of rectal pain associated with minimal rectal bleeding. On examination there is a small tear of the anorectal mucosa at the 6 o'clock position. Which of the following is the most appropriate initial treatment? Botulinum toxin Nitroglycerin Sitz baths, psyllium fiber, and bulking agents Sphincterotomy

Correct Answer ( C ) Explanation: This patient has classic findings for acute rectal fissure. Typically, the patient reports severe pain during a bowel movement, with the pain lasting several minutes to hours afterward. The pain recurs with every bowel movement, and the patient commonly becomes afraid or unwilling to have a bowel movement, leading to a cycle of worsening constipation, harder stools, and more anal pain. Approximately 70% of patients note bright-red blood on the toilet paper or stool. A fissure is a small cut or split in the anoderm. It may be induced by a hard bowel movement or straining at stool. Initially, the fissure is just a tear in the anal mucosa and is defined as an acute anal fissure if found within 3 months of onset. If the fissure persists over time, it progresses to a chronic fissure that can be distinguished by its classic features. A sentinel tag or "pile" is frequently observed protruding from the anus. The proximal end of the fissure may contain granulation tissue that is often confused with an anal polyp. The area around the fissure becomes sclerotic and appears white. Fissures are most commonly located anterior or posterior to the anus. When fissures are found laterally, syphilis, tuberculosis, occult abscesses, leukemic infiltrates, carcinoma, herpes, acquired immunodeficiency syndrome or inflammatory bowel disease should be considered as causes. Treatment for a fissure is quite simple when it is identified within three months of onset. First-line medical therapy consists of therapy with stool-bulking agents, such as fiber supplementation and stool softeners. Sitz baths after bowel movements and as needed provide significant symptomatic relief because they relieve some of the painful internal sphincter muscle spasm. Topical pharmacological agents that dilate the internal sphincter, including diltiazem, nifedipine, and nitroglycerin (B) are used for treatment of chronic anal fissures and also for acute anal fissures that have not responded to initial conservative therapy. Botulism toxin injections (A) and spincterotomy (D) should be conserved for patients with chronic anal fissures that do not respond to conservative or pharmacologic treatment. Botulism toxin is injected directly into the internal anal sphincter, in effect performing a chemical sphincterotomy. The effect lasts about 3 months, until nerve endings regenerate. If botulinum toxin injection provides initial relief of symptoms but there is a recurrence after 3 months, the patient may benefit from surgical sphincterotomy.

A 58-year-old woman presents with complaints of incontinence that has worsened over the past three months. She states that she has extreme urges to urinate, but is often unable to make it to the bathroom prior to leakage of urine. She notes that the incontinence is not related to coughing or sneezing. She states that behavioral interventions, such as voiding schedules, have not been helpful. Physical exam is unremarkable including presence of good rectal sphincter tone. Which of the following is the most appropriate intervention? Instruction in pelvic muscle exercises Intermittent urinary catheterization Oxybutynin Topical estrogen cream

Correct Answer ( C ) Explanation: This patient most likely has urge incontinence due to detrusor overactivity causing urinary leakage from bladder contractions. Urge incontinence is classified as urinary urgency and inability to delay urination. It is the most common cause of established geriatric incontinence and is usually idiopathic. The cornerstone of urge incontinence treatment is bladder training. If behavioral approaches have not been helpful, drug therapy with anticholinergic medications, such as oxybutynin or tolterodine, provide additional relief by relaxing the bladder. Pelvic muscle exercises (A) are most helpful in the treatment of stress incontinence which is due to relaxation of pelvic musculature. They are also helpful in urge incontinence; however, since the patient has been unsuccessful with behavioral modification, medication management is appropriate at this time. Intermittent urinary catheterization (B) is indicated in the treatment of overflow incontinence. Overflow incontinence is characterized by the involuntary release of urine from an overly full urinary bladder, often in the absence of any urge to urinate as may occur in paraplegics. This patient states she experiences strong urges to urinate, consistent with the diagnosis of urge incontinence, not overflow. Topical estrogen cream (D) is indicated in cases of stress incontinence when there is physical examination evidence of vaginal or urethral atrophy.

An 82-year-old woman presents for evaluation of pain on her sacrum. The patient is alert and oriented to person, place, and time. Physical examination is notable for a stage II decubitus ulcer. Multiple bruises are noted on her legs as well. She acknowledges that she remains in bed most of the day and sometimes her son who is her caretaker forgets to give her meals or change her diaper. He also squeezes her legs hard when she does not move quickly enough. You are concerned about elder neglect and abuse. She does not wish to report her son or speak with social work. Which is your most appropriate action? Call the son to discuss the case Discharge the patient home Notify adult protective services Psychiatry consultation for capacity determination

Correct Answer ( C ) Explanation: Up to two million elderly people are abused or neglected each year in the US. There are several categories of elder abuse including: physical abuse, sexual abuse, neglect, emotional or psychological abuse, abandonment, and financial or material exploitation. Unlike child abuse, interventions cannot be made against the abused persons wishes. Adults have the right to refuse intervention even if that means returning to a dangerous situation. However, in most states, emergency physicians are mandatory reporters of suspected elderly abuse or neglect and adult protective services should be informed. You cannot call the son (A) to discuss the case unless the patient gives you permission to do so. All physicians have the ability to determine capacity (D) and psychiatric consultation is not required for this process. In situations where it is unclear, they can be a useful resource. The patient may ultimately be discharged home (B) if she desires, but a clear follow-up plan should be in place. The physician may also consider admission to the hospital while the wound is evaluated and the case is discussed among the patient, adult protective services, and social work.

A 72-year-old man with metastatic prostate cancer presents with severe increasing lower back pain. His lower extremity strength is 4/5 and symmetrically weak. He has no bowel or bladder dysfunction and no perianal anesthesia. Which of the following is the most appropriate imaging test? Bone scan CT scan without contrast of the lumbar spine MRI of the thoracic and lumbar spines Radiographs of the lumbar spine

Correct Answer ( C ) Explanation: With a history of cancer and increasing low back pain, the most worrisome diagnosis is spinal cord compression. When cancer becomes metastatic to the spine, tumor can extend into the epidural space causing compression of the spinal cord. Additionally, tumors compress the venous plexus leading to inflammation and vasogenic edema. The most common cancers associated with cord compression are breast, prostate and lung. Classic symptoms include pain, motor deficits and bowel or bladder dysfunction. Bowel and bladder symptoms are late symptoms and their absence does not rule out the possibility of cord compression. Pain is the initial symptom in 95% of patients. The gold standard for diagnosis is MRI and should include both the lumbar and thoracic spine since there are often lesions at both levels. Bone scanning (A) is helpful in identifying bony metastasis of the entire skeleton in one imaging study. The radionuclide test identifies metastases with increased blood flow or new bone formation potentially missing disease without that (e.g. multiple myeloma). The study also does not give any information about the thecal sac necessary for the diagnosis of cord compression. CT scan without contrast of the lumbar spine (B) is helpful in the evaluation of the bony architecture of the spine and identification of metastatic disease but does not give information about the cord or thecal sac. Radiographs of the lumbar spine (D) may identify metastatic disease with less sensitivity than the other modalities mentioned. Metastatic disease is necessary for cord compression, however the lower sensitivity does not allow the sole evaluation with plain films.

A 2-year-old previously healthy boy presents with fussiness, rhinorrhea, and tugging on his right ear for the past day. Which of the following features is most sensitive for the diagnosis of acute otitis media? Absence of cough An erythematous tympanic membrane High-grade fever Impaired tympanic membrane mobility on pneumatic otoscopy

Correct Answer ( D ) Explanation: Acute otitis media refers to acute infection of the middle ear, and can be caused by bacterial, viruses, or fungi. The most common causes of bacterial infection in children are Streptococcus pneumoniae, nontypeable Haemophilus influenzae, and Moraxella catarrhalis. Common viral pathogens include respiratory syncytial virus, parainfluenza virus, rhinovirus, and adenovirus. Symptoms of acute otitis media are nonspecific and include ear pain or pulling on ears, fussiness, low-grade fever, cough, rhinorrhea, and vomiting and diarrhea. Physical exam features which support a diagnosis of otitis media include a tympanic membrane which is bulging and appears cloudy. Tympanic membrane hyperemia may be caused by crying or fever, but a deep red or hemorrhagic tympanic membrane is suggestive of acute otitis media. Air-fluid levels, bubbles behind the tympanic membrane, and an opacified tympanic membrane all indicate a middle ear effusion. Pneumatic otoscopy is helpful in confirming acute otitis media and impaired tympanic membrane mobility indicates a middle ear effusion and is one of the most sensitive indicators (96.5%) for acute otitis media. Absence of cough (A) is a Centor criteria supporting the diagnosis of Streptococcal pharyngitis, but it not relevant in the diagnosis of acute otitis media. An erythematous tympanic membrane (B) alone is not as sensitive for the diagnosis of acute otitis media as pneumatic otoscopy. High-grade fever (C) is not as sensitive for acute otitis media (40%).

A 30-year-old woman complains of anterior knee pain that gets worse with prolonged sitting, going up and down stairs and with deep squats. She has no known history of knee injury. She exercises three times a week on her exercise bicycle. She is average weight and played basketball as a high school student. Which of the following is the most likely diagnosis? Chronic anterior cruciate ligament tear Osgood-Schlatter disease Osteochondritis dissecans Patellofemoral syndrome

Correct Answer ( D ) Explanation: Patella-related pain is the single most common cause of knee pain. Patellofemoral pain syndrome is a multifactorial syndrome characterized by aching anterior knee pain that worsens with activities that stress the patellofemoral joint (climbing stairs, kneeling). Patients complain of diffuse, aching anterior knee pain that is exacerbated by loaded flexion activities such as stair climbing, jumping, or prolonged sitting (theater sign). On exam there may be patellar crepitation. The diagnosis is clinical and knee radiographs provide limited information (articular cartilage loss). Treatment includes activity modification and an exercise program consisting of quadriceps strengthening (medial quadriceps) and hamstring flexibility. NSAIDs are recommended for symptomatic care. Osteochondritis dissecans (C) is due to repetitive small stress to the subchondral bone that leads to osteonecrosis (most commonly the medial femoral condyle). Most cases begin in childhood although individuals may not become symptomatic until late adolescence or early adulthood. Early diagnosis is critical, as the injury has a better potential to heal while the bones are still growing. Osgood-Schlatter disease (B) is caused by rupture of the growth plate at the tibial tuberosity, which causes stress on the patellar tendon. It most commonly occurs in rapidly growing adolescents (10-15-years-old) and is five times more common among those active in sports and up to three times more common in boys. Chronic anterior cruciate ligament (ACL) tear (A) is a chronic injury seen in individuals who have a history of recurrent episodes of knee instability associated with swelling and limited motion. Patients may describe locking or a "giving way" phenomenon. They may also give a history of a remote injury to the knee that was not rehabilitated.

A 12-month-old girl presents with recurrent urinary tract infections. The external genital examination is normal. Renal ultrasonography shows hydronephrosis. Voiding cystourethrography does not reveal any detrusor abnormality. Which of the following is the most likely diagnosis? Bladder diverticulum Cryptorchidism Epispadia Vesicoureteral reflux

Correct Answer ( D ) Explanation: Retrograde passage of urine from the urinary bladder into the ureter is called vesicoureteral reflux (VUR), a condition which causes urinary tract infection in 30-40% of infants and toddlers. Causes are many, but most VUR is caused by a congenital abnormality of ureterovesicular junction, particularly a defect in the normal flap-valve mechanism in the intramural ureteral canal. An upper tract bacterial infection is a common result. Over time, recurrent kidney infections can lead to renal scarring, dysfunction and systemic hypertension in the adult population. Diagnosis is made by ultrasonography and voiding cystourethrography (VCUG). Symptoms may resolve spontaneously over months to years. Treatment may include prophylactic antibiotics and ureteral re-implantation. Bladder diverticulum (A) is a herniation of bladder mucosa through a defect in the bladder muscle (detrusor). It can occur with VUR. A VCUG would show results of an abnormal bladder wall. Cryptorchidism (B) refers to undescended testes, which are not found in girls. Epispadia (C) is when the urethral meatus opens on the dorsum of the glans penis, the penile shaft or between the clitoris and labia. This can be found during external genital inspection.

A 27-year-old woman develops back pain, diffuse joint pain, headache, and dyspnea shortly after starting a blood transfusion for heavy vaginal bleeding. Vital signs are notable for fever, tachycardia and hypotension. Which of the following is the most important initial treatment? Administer antipyretics Administer IV fluids Repeat type and screen and cross-matching, serum haptoglobin, complete blood count and direct Coombs testing Stop transfusion

Correct Answer ( D ) Explanation: The patient developed an acute hemolytic reaction and the most important priority is stopping the transfusion. Acute hemolytic reactions are a medical emergency and occur from the rapid hemolysis of donor red blood cells from host antibodies. Symptoms occur within minutes of onset of the transfusion and include fever, chills, and back and joint pain. Uncontrolled hemolysis can lead to acute renal failure and DIC. The majority of acute hemolytic reactions occur because of transfusion of ABO incompatible blood (i.e. type A red cells transfused to a type O individual), usually due to human error. Antipyretics (A) is inadequate treatment for an acute hemolytic reaction, which is life-threatening. Although IV fluids (B) are an important part of the management of acute hemolytic reaction, the first priority is stopping the transfusion. Similarly, repeat type and screen and cross-matching, serum haptoglobin, complete blood count and direct Coombs testing (C) should be performed to confirm the diagnosis of an acute hemolytic reaction, but again the most immediate priority is stopping the transfusion.

A 28-year-old woman complains of sudden onset of lower back pain that began two days ago. The pain radiates down both legs. On exam, she has 3/5 strength bilaterally, decreased sensation in her lower extremities, decreased patellar reflexes, and a distended urinary bladder. An MRI of her spine reveals no masses. Which of the following is the most likely diagnosis? Guillain-Barré Syndrome Spinal epidural abscess Syringomyelia Transverse myelitis

Correct Answer ( D ) Explanation: Transverse myelitis is an acute spinal cord disorder characterized by paraplegia, a transverse level of sensory impairment, and sphincter disturbance. The specific etiology is unknown, although it is noted to follow a viral infection in 30% of patients and is commonly referred to as postinfectious myelitis. Patients often complain of back pain and may have a low-grade fever, raising the concern for an epidural abscess. An MRI is usually performed to rule out compressive lesions of the spine. Despite the absence of evidence, most sources still recommend high dose steroids as first line treatment. Although spinal epidural abscess (B) has many clinical similarities to transverse myelitis, the population at risk for this diagnosis is generally limited to individuals with a history of intravenous drug abuse or prior spinal instrumentation. Guillain-Barré syndrome (A) is an acute demyelinating disorder of the peripheral nerves. It is classically associated with an antecedent viral illness followed by the onset of ascending, symmetric paresthesias, and motor weakness. Unlike transverse myelitis, GBS is less commonly associated with back pain and usually occurs over days to weeks whereas transverse myelitis occurs more acutely. Syringomyelia (C) results from a CSF fluid collection within the spinal cord. Any location is possible, but it is most commonly seen in the cervical spine in association with Arnold-Chiari malformation. An MRI will confirm the diagnosis.

After your evaluation of a young girl's complaints, you suspect an endocrinopathy is present. Which of the following symptoms points toward pituitary gland pathology? Abdominal pain Hypertrophic cervical lymphadenopathy Pelvic pain Visual disturbance

Correct Answer ( D ) Explanation: Tumors of the pituitary gland are common neoplasms, representing 10-15% of all intracranial tumors. Pituitary tumors can be characterized by size (microadenoma versus macroadenoma), functionality (hormone excess versus hormone deficiency) and immunohistochemical abnormality. When evaluating a suspected endocrinopathy, visual field testing is a necessary physical exam step. Visual field abnormalities, most commonly bitemporal hemianopia, are the most common neurologic findings associated with optic chiasm deformity. Neoplasia of the pituitary gland, which is seated directly beneath the optic chiasm, causes a mass effect on the chiasm, resulting in the most common presenting symptom of a pituitary tumor, visual disturbance. Nonspecific headache can also occur, as well as diplopia, ptosis, Horner's syndrome and spontaneous cerebrospinal fluid rhinorrhea. Abdominal pain (A) is associated with adrenal, not pituitary, gland tumors. Hypertrophic lymphadenopathy (B) of the neck points toward a thyroid, not pituitary, gland tumor. Pelvic pain (C) points toward a sex-organ etiology, not a pituitary etiology.


Related study sets

BUS 3010: Chapter 17 - Managing Conflict, Politics, and Negotiation

View Set

Intro to Cell- Final Exam w/Coach Moon

View Set

AH2- Blood Administration Module Quiz A F17- ATI

View Set

Testout Study Guide Chapters 2-4

View Set

CMA Part 2 Section D- Risk Management

View Set